Business Accounting Vol 5 1920
Business Accounting Vol 5 1920
Business Accounting Vol 5 1920
OF
THE UNIVERSITY
OF CALIFORNIA
LOS ANGELES
VH, Y OF CALIFORNIA,
-iZiRARY,
LDS ANGELES, CALIF.
BUSINESS ACCOUNTING
Harold Dudley Greeley, C.P.A., Editor
II —Constructive Accounting
By George E. Bennett
A VOLUME V
ILLUSTRATIVE ACCOUNTING
PROBLEMS
By
CHARLES F. RITTENHOUSE, B.C.S.
Certified Public Accountant; Professor of Accounting, Boston
University; Member of American Institute of Accountants;
Member of National Association of Cost Accountants
and
Third Prinling
NEW YORK
THE RONALD PRESS COMPANY
1921
40010
Copyright, 1920, by
HF
EDITORIAL PREFACE
Ten years ago almost any contribution to the liter'
April 1, 1920
Illustrative Accounting
Problems
Illustrative Accounting Problems
Problem 1
337 $200.00
351 400.00
352 750.00
355 586.00
356 182.00
357 241.00
360 346.00
361 150.00
362 125.00
363 294.00
Required
Statement reconciling cash book and pass-book.
Solution to Problem 1
RECONCILIATION
Balance, per pass-book $10,010.00
Add— Checks not credited 3,264.00
$13,274.00
ILLUSTRATIVE PROBLEMS
Deduct — Checks outstandiog
337 $200.00
351 400.00
352 750.00
355 586.00
356 182.00
357 241.00
360 346.00
361 150.00
362 125.00
363 294.00. 3,274.00
Problem 2
Receipts
1912
Jau. 4 Collections from custoi 2,818.62
7 1,147.33
10 1,064.87
13 1,232.55
16 1.463.24
23 2.417.14
26 1,283.84
29 1,543.62
31 1,054.27
$13,925.48
(Ohio C. P. A. Examination.)
6 ILLUSTRATIVE PROBLEMS
Solution to Problem 2*
Exhibit A
BANK STATEMENT AND RECONCILIATION
1912
Jan. 1 Debit balance 0/D $ 10.32
31 Debit total checks cashed 8,832.43 $ 8,842.75
Exhibit B
RECEIPTS, DEPOSITS, AND DISBURSEMENTS
Receipts and Deposits:
January cash book receipts $14,025.48
—
Less Cash not deposited 56.23
Bank Disbursements:
Actually paid in January $8,832.43
Checks outstanding as issued 53.27
•Solution by W. P. Kohr. C. P. A.
ILLUSTRATIVE PROBLEMS 7
Exhibit C
CASH BOOK—STATEMENT AND CORRECTIONS
1912
Jan. 31 Receipts per footings $13,925.48
Add — To correct error in footing 100.00
$14,025.48
1 Balance overdrawn $ 10.32
31 Disbursements, per footing . . . $8,885.34
Deduct— To correct error .... 10.00 8,875.34
Problem 3
Solution to Problem 3
Problem 4
Solution to Problem 4
Problem 5
Solution to Problem 5
ter XXIV, § 4.
Problem 6
Solution to Problem 6
Problem 7
*
Solution to Problem 7
Solution by E. P. Morey, C. P. A.
14 ILLUSTRATIVE PROBLEMS
B and Company $1,000.00
Sales $1,000.00
Sold bill of goods 2/10, net 30.
Cash 333.34
Notes Receivable 666.66
B and Company 1,000.00
Received cash and notes in settlement of account.
Cash 662.49
Bank Discount 4.17
Notes Receivable Discounted 666.66
Discounted notes of B and Company at bank. Rate
of discount, 5%.
Cash 134.33
Notes Receivable 200.00
Notes Receivable 333.33
Bank Discount 1.00
Received from B and Company cash and new note
in renewal of their 90-day note.
Problem 8
Solution to Problem 8
Chapter XXIX, § 4.
(b) Replacements. The bookkeeping to record replacements is
Volume I, Chapter XXIX, § 5.
discussed in
(c) Surplus Adjustment. The necessity for adjusting surplus,
capital, undivided profits, or other similar accounts to
record items applicable to prior fiscal periods is discussed
in Volume IV, Chapter VII, § 4.
Problem 9
Solution to Problem 9
Depreciation $ 900.00
Reserve for Depreciation $ 900.00
To set up the 10th and last annual reserve for
depreciation.
Cash 2,000.00
Machinery 2,000.00
To show sale of old machine for cash.
Machinery 15,000.00
Cash 15,000.00
To show purchase of new machinery.
Problem 10
Solution to Problem 10
Surplus 17.50
Reserve for Depreciation of Delivery Equipment . . 17.50
To correct a previous entry charging $100 to reserve
when only $82.50 had been reserved. Cost of horse
$275; sold for $175; three years' depreciation at 10%
$82.50.
Problem 11
Solution to Problem 11
Cash 127.64
Reserve for Bad Debts 191.46
Accounts Receivable 319.10
Final dividend of 20% from trustees of Thomas Knight
—claim $638.20, previous dividends, 30% and 20%.
Balance 30% charged off.
J. C. Cutter 42.25
Surplus 42.25
Cutter pays amount previously written off; above en-
try is to reopen his account.
Cash 42.25
J. C. Cutter 42.25
Received of J. C. Cutter amount previously written off
as uncollectible.
(a) Provision for Bad Debts. In this problem the provision for
bad debts was based on the amount of accounts receivable
outstanding. It is more customary to base this provision
ILLUSTRATIVE PROBLEMS 21
Problem 12
Solution to Problem 12
ADJUSTMENTS
June 30, 1920
Surplus $1,712.50
Inventory $1,712.50
To adjust error in including two lathes billed to the
Essex Machine Company in the inventory of
December 31, 1919.
Surplus 2,100.00
Inventory 2,100.00
To adjust clerical errors in figuring inventory of
December 31, 1919:
Finished stock $3,100 too much
Raw materials 1,000 too little
Cash 300.00
Accounts Receivable 300.00
L. P. Fuller on account.
Cash 300.00
Accounts Receivable 300.00
Problem 13
Sales $45,000.00
Purchases 30,000.00
Freight and Cartage (on sales $500, on purchases $500) 1,000.00
Returns by Us 2,500.00
Cash Discounts Allowed Customers 6,750.00
Cash Discounts Taken by A 750.00
Allowances to Customers 1,000.00
Inventory at the close of the year is valued at 5,000.00
Solution to Problem 13
Merchandise
Purchases $30,000.00
Freight and Cartage In 500.00
Freight and Cartage Out 500.00
Cash Discounts on Sales 6,750.00
Sales Returnsand Allowances 1,000.00
Merchandise 9,500.00
Sales $45,000.00
Purchase Returns and Allowances 2,500.00
Cash Discounts on Purchases 750.00
The above entry is to close out the Merchandise
account and to set up instead separate ac-
counts showing the various trading activities.
STATEMENT OF PROFIT
Sales
Gross Sales $45,000.00
Deduct — Returns and Allowances 1,000.00
JOURNAL ENTRIES
Sales $ 1,000.00
Sales Returns and Allowances $ 1,000.00
To show net sales in Sales account.
Purchases 500.00
Freight and Cartage In 500.00
To show total cost of purchases in Purchases ac-
count.
Sales 44,000.00
Trading Account 44,000.00
(g) Freight and Cartage Outward. This item does not appear in
the Trading account because it is not an expense to be
considered in determining the gross profit. For a discus-
sion of it see Volume I, Chapter IX, § 7.
(h) Bookkeeping to Facilitate Statement Preparation. The
accounts are stated in this solution in a way to facilitate the
preparation of financial statements. The desirability of
this is pointed out in Volume I, Chapter VI, § 9.
(i) Account for Merchandise Inventory. A separate account
is opened for the merchandise inventory, as suggested in
Problem 14
Solution to Problem 14
First Solution
Second Solution
Problem 15
Draper Monroe
February 23 $ 600.00 January 25 $1,000.00
May 26 1,000.00 March 1 500.00
October 15 900.00 September 1 2,000.00
Solution to Problem 15
(a)
$78.03
so ILLUSTRATIVE PROBLEMS
H. M. Monroe, Current Account 121.00
Interest on Capital 121.00
Interest on drawings at 6% as follows:
$1,000 for 11 mos. 6 days $ 56.00
500 " 9 " 30 " 25.00
2,000 " 3 " 30 " 40.00
$121.00
Total $985.05
Credit:
Interest on Capital 925.63
Debits:
Interest on Drawings $121.00
One-third of debit balance of Interest Account 453.51
Total 574.51
(b)
Problem 16
Solution to Problem 16
A, Capital 206.25
Interest on Capital 206.25
To charge A with interest on drawings at 5% as
follows: Drawings of $9,000 for the year on the
last day of each month amount to a monthly
drawing of $750; the first drawing would be
charged interest for 11 months, the second, 10
months, etc., making an interest charge on $750
for a total of 66 months or 5j^ years, or $206.25.
B, Capital 187.50
Intereston Capital 187.50
As B drew $2,500 on the last day of each quarter,
he would be charged interest on the first drawing
for 9 months, on the second for 6 months, on the
third for 3 months, making a total of 18 months'
interest charge on $2,500 at 5%, or $187.50.
A
Investment $30,000.00
Less— Drawings 9,000.00
Add:
Interest on Capital $ 1,500.00
One-half Net Profit 13,196.87 $14,696.87
B
Investment $50,000.00
Less— Drawings 10,000.00
Add:
Interest on Capital $ 2,500.00
One-half Net Profit 13,196.88 $15,696.88
Solution to Problem 17
$37,500.00 $37,500.00
Chapter XXXII, § 6.
(c) Offsetting Loan with Deficit. As explained in Volume I,
Problem 18
Solution to Problem 18
$70,000.00 $70,000.00
Problem 19
Solution to Problem 19
Problem 20
Solution to Problem 20
stalment.
This does not yet leave the accounts standing in the
ratio of 50, 30,and 20, Z's account being proportionately
less; since the purpose is to establish the above ratio as
soon as possible, the second instalment is treated the
same as the first. After it is ready for distribution the
remaining assets of $35,467.50 are treated as a potential
loss shared, X $17,733.75, Y $10,640.25, and Z $7,093.50,
leaving X a credit balance of $41,384.88, Y $24,830.92,
and Z $8,406.50, which is the basis for distribution of
second instalment.
ILLUSTRATIVE PROBLEMS 41
$95,000.00 $95,000.00
$48,000.00 $48,000.00
$17,000.00 $17,000.00
4« ILLUSTRATIVE PROBLEMS
Problem 21
Solution to Problem 21
$15,250.00 $15,250.00
ILLUSTRATIVE PROBLEMS 43
$3,875.00 $3,875.00
Interest on Capital
$1,125.00 $1,125.00
A Capital
$10,750.00 $10,750.00
Balance $ 8,812.50
B Capital
$5,375.00 $5,375.00
Balance $4,212.50
C Salary Account
2/10 Net Loss $775.00 Salary due $250.00
Balance . . 525.00
$775.00 $775.00
Balance $525.00
Problem 22
$232,820.57 $232,820.57
Solution to Problem 22
(a)
(c)
(b)
JONES
WORKING
Six Months Period, January 1,
Land $ 55.000.00
Building 37,500.00
Furniture and Fixtures. 6,820.00
Cash. 7,682.53
Accounts Receivable 23,731.40
Notes Receivable 730.00
Inventory 24,260.75
Mortgage Payable 35,000.00
Accounts Payable 9,840.62
Notes Payable 5,000.00
C. R. Ross, Salary. 250.00
Reserve for Depreciation of Building. 7,500.00 (6) $375.00
Reserve for Depreciation of Furn. and Fixtures. 1,750.00 (7) 291.00
Reserve for Loss on Bad Accts. and Notes Rec 169.80 (8) 413.44
H. B. Jones, Capital 60,000.00
H. B. Jones, Drawings 1,869.00
C. R. Ross, Capital 30,000.00
C. R. Ross, Drawings 4,705.00
Sales. 82,687.10
Purchases 53,321.60
Freight, Express, and Cartage In. 1,924.34
Traveling Expenses 2,107.40
Salaries and Wages 9,369.72
Delivery Expenses 1,290.81
Office Expenses 1,587.10 (5) 150.89
Insurance 435.00 (1) 260.00
Intereston Notes Receivable. . . . 186.24 (3) 24.60
Intereston Notes Payable 238.90 (4) $75.00
Intereston Mortgage Payable . . . 875.00
Cash Discounts on Purchases .... 486.72
Cash Discounts on Sales 372.02
$232,820.57 $232,820.57
$1,692.43 $1,692.43
ILLUSTRATIVE PROBLEMS 51
AND ROSS
SHEET
1920, to June 30, 1920
1 % 55,000.00 $ 55,000.00
9. 37,500.00 37,500.00
3 5,820.00 5,820.00
4 7,682.53 7.682.53
5 23,731.40 23,731.40
6 730.00 730.00
7 24,260.75 $ 24,260.75 $ 25,710.40 25,710.40
8 $ 35,000.00 $ 35,000.00
9 9,840.62 9 840 62
in 5,000.00 5,000 00
11 250.00 250.00
1"? 7,875.00 7,875 00
13 2,041.00 2,041.00
14 583.24 583.24
15 60,000.00 60,000.00
16 1,869.00 5,498.40 3,629.40
17 30,000.00 30,000.00
18 4,705.00 4,705.00
19 82.687.19 3,421.70 86.108.89
90 53,321.60 57.529.46 4,207.86
9\ 1,924.34 $ 1,924.34
2? 2,107.40 2,107.40
93 9,369.72 9,369.72
24 1,290.81 1,290.81
25 1,436.21 1,436.21
?n 175.00 175.00
27 160.84 $ 160.84
98 313.90 313.90
99 875.00 875.00
30 486.72 486.72
SI 372.02 372.02
39 260.00 260.00
33 102.50 102.50
34 102.50 102.60
35 24.60 24.60
36 75.00 75JM)
37 150.89 150.89
38 375.00 375.00
39 291.00 291.00
40 413.44 413.44
$234,102.11 $234,102.11
$116,027.15 ni6,027.I6
(d)
Assets
Fixed Assets:
Land (cost) $55,000.00
Building (cost) $37,500.00
Less— Reserve for Depreciation . . 7,875.00 29,625.00
Current Assets:
Cash on Hand $ 7,682.53
Accounts Receivable $23,731.40
Notes Receivable 730.00
$24,461.40
Less — Reserve for Bad Debts and
Notes Receivable 583.24 23,878.16
Fixed Liabilities:
Mortgage Payable $ 35,000.00
Current Liabilities:
Accounts Payable $ 9,840.62
Notes Payable 5,000.00
Accrued Interest on Notes Payable 75.00
ILLUSTRATIVE PROBLEMS 53
$11,907.04
Less — Drawings, January 1, 1920,
to June 30, 1920 5,498.40 6,408.64 66,408.64
(e)
Trading $24,260.75
Inventory $24,260.75
Goods on hand January 1, 1920, per inventory.
Trading 53,321.60
Purchases 53.321.60
Net purchases for six months ending June 30,
1920.
Sales 82,687.19
Trading 82.687.19
Net sales for six months ending June 30. 1920.
54 ILLUSTRATIVE PROBLEMS
Inventory 25,710.40
Trading 25,710.40
Goods on hand June 30, 1920, per inventory.
Trading 30,815.24
Profit and Loss 30,815.24
To transfer to Profit and Loss account the gross
trading profit for the six months ending June
30, 1920, as represented by the balance of the
Trading account.
413.44
To transfer to Profit and Loss account the bal-
ances of the accounts representing extraneous
expenses for the six months ending June 30,
1920.
ILLUSTRATIVE PROBLEMS 55
Problem 23
BALANCE SHEET
December 1, 1919
$180,000.00 $180,000.00
Solution to Problem 23
Cash $ 5,000.00
Accounts Receivable 30,000.00
Inventory 120,000.00
Sundry Assets 25,000.00
Good- Will 25,000.00
Smith, Jones and Company $205,000.00
To bring onto the books the assets acquired
from Smith, Jones and Company.
Problem 24
Solution to Problem 24
June 1
Problem 25
Following is the liabilities and capital section of the
balance sheet of a corporation:
Capital Liabilities:
Preferred Stock $100,000.00
Common Stock 500,000.00
First Mortgage Bonds 100,000.00 $ 700,000.00
Current Liabilities:
Notes Payable $ 40,000.00
Acceptances Payable 120,000.00
Accounts Payable 10,000.00 170,000.00
Reserves:
Depreciation $ 20,000.00
Bad Debts 5,000.00
Insurance 10,000.00
Contingencies 20,000.00 55,000.00
Surplus 136,418.20
Total $1,061,418.20
ILLUSTRATIVE PROBLEMS 61
Solution to Problem 25
Net Worth of Company:
Preferred Stock $100,000.00
Common Stock 500,000.00
Surplus 136,418.20
Insurance Reserve 10,000.00
Contingent Reserve 20,000.00
$766,418.20
Deduct— Preferred Stock Equity 100,000.00
Problem 26
The Nassau Engineering Company fails and a re-
ceiver is appointed on March 1, 1920, who on taking
Solution to Problem 26
March 1, 1920
Assets
Expected to
Book Value Realize Deficiency
Expected to
Book Value Realize Deficiency
22,500.00 Securities Owned 1,500.00
$14,000 pledged with
fully secured creditors
having claims of ... . $13,500.00
Pledged with partly se-
cured creditors 7,500.00
\ $21,000.00
$30,000.00
$ 2,000.00
eating a dividend to
creditors of approxi-
mately 85%) $59,034.00
Deficiency, per Deficiency
Account 10,566.00
$69,600.00
64 ILLUSTRATIVE PROBLEMS
Liabilities
Expected to
Book Value Rank
$ 89.100.00 Accounts Payable $68,100.00
Deducted contra $21,000.00
10,000.00 Mortgage Payable
Deducted contra 10,000.00
Notes Receivable Dis-
counted 1,500.00
Unpaid Taxes and Assess-
ments
Deducted contra 216.00
Capital Obligations:
Capital Stock $75,000.00
Less— Deficit 48,850.00
$125,250.00
$85,566.00 $85,566.00
ILLUSTRATIVE PROBLEMS 65
Problem 27
TRIAL BALANCE
At June 30, 1914
Real Estate and Buildings $125,000.00
Capital Stock $300,000.00
Machinery and Equipment 160,000.00
Customers' Accounts Receivable 170,000.00
Notes Payable 250,000.00
Accounts Payable 312,000.00
Insurance Premiums Unexpired 3,000.00
Mortgage on Buildings 65,000.00
Notes Receivable 26,000.00
Interest Accrued on Mortgage 2,500.00
Cash on Hand and in Bank 6,500.00
Inventory of Raw Material 85,000.00
Inventory of Finished Goods 121,000.00
Investments 12,000.00
Deficit 221,000.00
$929,500.00 $929,500.00
Solution to Problem 27
STATEMENT OF AFFAIRS
At June 30, 1914
Assets
Estimated to
Book Value Realize
$17,500.00
Less — Notes held to secure Notes
Payable:
Book Value $15,000.00
Will Realize 10.000.00 10,000.00 7,500.00
68 ILLUSTRATIVE PROBLEMS
Estimated to
Book Value Realize
Finished Goods 121,000.00 $121,000.00
Less — Pledged to secure Account
Payable 16,000.00 105,000.00
De6cit 221,000.00
Total $929,500.00
$519, ''00.00
Liabilities
Estimated
Book Value to Rank
Preferred Claims (none)
Fully Secured Claims:
Mortgage on Buildings $ 65,000.00
Mortgage
Interest on 2,500.00
Deducted from Assets (per contra)
Partly Secured Claims:
Notes Payable 250,000.00
Less —
Notes Receivable (de-
ducted per contra) $ 10,000.00 $240,000.00
Estimated
Book Value to Rank
Unsecured Claims:
Accounts Payable 257,000.00 257,000.00
Capital Stock 300,000.00
Total $929,500.00
DEFICIENCY ACCOUNT
Book Deficit $221,000.00 Capital Stock $300,000.00
Shrinkage in Accounts Appreciation of Invest-
Receivable 58,000.00 ments 4,500.00
Shrinkage in Insurance Net Deficiency, per
Premium Unexpired . 800.00 Statement of Affairs 32,800.00
Shrinkage in Notes Re-
ceivable 8,500.00
Shrinkage in Machinery
and Equipment .... 25,000.00
Shrinkage in Real
Estate 24,000.00
$337,300.00 $337,300.00
Problem 28
$769,625.00 $769,625.00
Solution to Problem 28
ADJUSTING ENTRIES
October 31, 1913
Purchases 1,800.00
Accounts Payable 1,800.00
To enter on the books invoices for purchases which
were omitted.
Sales $516,875.00
Deduct —
Cost of Sales:
Purchases $198,175.00
Add— Difference in Inventories . 27,525.00 $225,700.00
Deduct
Selling Expenses $ 70,500.00
Administrative and All Other Expenses 30,525.00 101,025.00
s
§
•E3
s s
en
s
T3
.2
"S cS
V V
tf
»
«3 o ©
o
o
tf d d
o <= 2
t— o
C/2 -<3
p^ i-h" »f
P^
^ ptH
O
i-s
<1 o
Pl^
Q O rH*^ ^
1^
w pq o
H ^ o
O
iX!
<
H
cfi
tf
bo
si
>> 60
-a
2 o 4) cS aj
CO
e8 "tJ
3 CO a
3 HH Ji
-3 CO
o « =S
"c 2a
3 —«
0) u
4)
^ -3 u I
§2 SOS
1 ioQ^ go J Q^ .5S
=
u
33
hJ ff
« O O
o
— q o
>c d »o o oo o
e8
> t- o (X o »fl o
CO O "O
-S 00
ILLUSTRATIVE PROBLEMS 75
«5 «5
<3* m
96-
o o o 9 ^
^ 2 JO 2
o o o
?o o
»
H
J2 m
Q -g
i> N -o
*"^
-s Q -2
0) o *-> CS
a;
ti
9 T3 >>
-i
-d
a ^ in
en
T3
c3
to r-) *j
o tH
c
05
o 3
-a Ji
cd
Ph
a 03
§ TS «
fc4
3
•— 05
t^ ei
3 ai a
*j
«
3 3
loo
1
iJ
cc 3
tn
4>
O 3
1
4>
V 'S a
C J3 .**
o u
C to .2 O 4>
s a 12;
a O
(^ 'o
a-o 1-' ca
« o V
1^ X v
t-,
eS I-]
3 o o
H i 1-9 1^ Q ;?; Q
76 ILLUSTRATIVE PROBLEMS
T3 Jd
U et
o o o
q q q
O «5
O
«s
l-H
o 00
t- o»
O* 1> of
CO « »-^
a& 09-
1^
V OS
3a is
a
>, o
08 W
(4/ ^ TV V a
tj) 60
60 -S
-J a 0}
a Q
_ a .
0) aw en
>. '^ U C8
e8 -a CO (^ S 08 U
P-iV a U <"
V o I— I _
OS -a
t« A) 01 N W "•s
!-> ja s
s ns
la a o •g hj I?: ^ h^fe^.«
ta^ OO
< PLi
ca
ILLUSTRATIVE PROBLEMS 77
(b)
Deficiency Account
$106,985.00 $106,985.00
Problem 29
PETER POST
REALIZATION AND LIQUIDATION ACCOUNT
$118,835.00 $118,835.00
Total $80,012.00
Closing Inventories:
Raw Material $ 5,000.00
Finished Goods 27,900.00 32,900.00
Net 47,112.00
Receipts:
Loan from Principal Creditors $ 6,000.00
Accounts Receivable:
Collections on Old Accounts 8,682.00
Collections on Trading Accounts 68,918.00 78,600.00
Payments:
Labor $15,725.00
Expenses 5,430.00
Machinery and Tools 750.00
Personal Drawings by Peter Post 3,500.00
Loan from Principal Creditors 6,000.00
Trade Creditors:
Payments on Old Accounts 25,289.00
Payments on Trading Accounts 21,211.00 77,905.00
Total $37,242.00
Fixed Assets:
Machinery and Tools 32,247.00
Total $69,489.00
Liabilities
Current Liabilities:
Trade Creditors $ 89.00
Total $73,280.00
Deduct
Loss on Realization of Assets $ 380.00
Cash Drawings by Peter Post .... 3,500.00 3,880.00
Total $69,489.00
Chapter VIII, § 7.
ILLUSTRATIVE PROBLEMS 83
Problem 30
Assets
$435,450.00
Liabilities
$435,450.00
84 ILLUSTRATIVE PROBLEMS
Receipts
Payments
Notes Payable $ 25,000.00
Accounts Payable 35,000.00
Interest on Mortgage, one year at 5% 5,000.00
Taxes for year 1907 865.00
Purchase of Material and Supplies 98,000.00
Labor 135,000.00
General Expenses 45,000.00
Interest on Bills Payable to September 30, 1908. at 5% 2,800.00
1908 $8,000.00
Bad debts written off accounts subsequent to Janu-
ary 1, 1908 2,000.00 10,000.00
Solution to Problem 30
X, Y, AND Z
Assets to be Realized
Land and Buildings $125,000.00
Machinery and Tools 75,000.00
Furniture and Fixtures 10,000.00
Materials and Supplies 95,000.00
Notes Receivable 15,000.00
Accounts Receivable 115,000.00 $435,000.00
Liabilities Liquidated:
Notes Payable $ 25,000.00
Accounts Payable 35,000.00
Interest Accrued on Mortgage 1,250.00
Taxes Accrued (estimated) 835.00 62,085.00
Supplementary Charges:
Taxes in Excess of Estimated Amount 30.00
$777,115.00
Liabilities to be Liquidated:
Mortgage Payable $100,000.00
Notes Payable 135,000.00
Accounts Payable 105,000.00
Interest Accrued on Mortgage 1,250.00
Taxes Accrued (estimated) 835.00 $342,085.00
86 ILLUSTRATIVE PROBLEMS
Assets Realized:
Notes Receivable $ 15,000.00
Accounts Receivable 106,500.00
Materials and Supplies, per Trustee's Trading
Account 95,000.00 216,500.00
AssetsNot Realized:
Land and Buildings $125,000.00
Machinery and Tools 75,000.00
Furniture and Fixtures 10,000.00 210,000.00
$777,115.00
X, Y, AND Z
Sales $340,435.00
Total $328,000.00
Inventory, December 31, 1908, per Balance
Sheet 92,000.00 236,000.00
Expenses:
General Expense $ 45,000.00
Interest on Mortgage 5,000.00
Interest on Notes Payable 2,800.00
Bad Debts Written Off 2,000.00
Taxes (estimated) 865.00
Discounts and Allowances 300.00 55,965.00
X, Y, AND Z
BALANCE SHEET
December 31, 1908
Assets
Fixed Assets:
Land and Buildings $125,000.00
Machinery and Tools 75,000.00
Furniture and Fixture:^ 10,000.00 $210,000.00
Current Assets:
Cash $ 6,220.00
Accounts Receivable 100,700.00
Notes Receivable 6,500.00
Merchandise 92,000.00 205,420.00
Current Liabilities
Notes Payable $180,000.00
Interest Accrued on Mortgage 1,250.00
Taxes Accrued 865.00 182,115.00
Capital
Balance, January 1, 1908 $ 93,365.00
Profit on Trading 48,470.00
$141,835.00
Loss on Realization and Liquidation, per Reali-
zation and Liquidation Account 8,530.00
Problem 31
Solution to Problem 31
Cash $ 850.00
Accounts Receivable 6,941.00
Merchandise 9,241.00
Real Estate 3,000.00
Store Fixtures 571.00
Liabilities
Net Profit:
Smith, one-half $1,831.50
Murray, one-half 1,831.50 $ 3,663.00
90 ILLUSTRATIVE PROBLEMS
OPENING ENTRIES
Cash $ 850.00
Accounts Receivable 6,941.00
Merchandise 9,241.00
Store Fixtures 571.00
Real Estate 3,000.00
Notes Payable $ 975.00
Accounts Payable 4,175.00
Smith, Capital 8,231.50
Murray, Capital 7,221.50
Cash 3,000.00
Notes Receivable 1,000.00
Merchandise 2,000.00
Davis, Capital 6,000.00
Problem 32
On January 1, 1920, Robert A. Grant began business
as a retail dry goods merchant. His capital at the
time consisted of the following assets: merchandise
$12,300, cash $1,150, furniture and fixtures $600.
There were no liabilities to be recorded at the outset
of the business enterprise.
He sold most of his goods for cash, although credit
was extended in certain cases.
ILLUSTRATIVE PROBLEMS 91
(a)
ROBERT A. GRANT
COMPARATIVE STATEMENT OF ASSETS AND
LIABILITIES
For Three Months
Assets
Jan. 1 Mar. 31 Increase
Liabilities
(b)
March 31, 1920
'
Problem 33
Received
Accounts Receivable $30,000.00
Capital Paid In 2,500.00
Disbursed
Bank Overdraft, January 1 $ 3,700.00
Accounts Payable 12,500.00
General Expense 5,000.00
Wages 7,750.00
Personal Account 1,500.00
Leaving a bank account of $2,000, and currency on hand $50.
Solution to Problem 33
$20,265.00
Less:
Capital Paid In $2,500.00
5% Interest on Capital of January I 1,725.00 4,225.00
Less —
Cost of Goods Sold:
Merchandise on Hand, January 1 $ 8,500.00
Net Purchases 14,750.00
$23,250.00
Less— Merchandise on Hand, December 31 9,500.00 13,750.00
Deduct:
General Expenses $ 5,000.00
Wages 7,750.00
Depreciation of Plant and Machinery 1,000.00
Depreciation of Furniture and Fixtures 35.00
Loss on Bad Debts 2,200.00
Interest on Capital (5% on $34,500) 1,725.00 17,710.00
Assets
Cash $ 2,050.00
Accounts Receivable $44,000.00
Less— Reserve for Bad Debts 2,200.00 41,800.00
Net Worth:
Capital, January 1 $34,500.00
Add:
Net Profit for the year $16,040.00
Capital Paid In 2,500.00
5% on Capital of January 1 1,725.00 20,265.00
$54,765.00
Less— Drawings 1,500.00 53,265.00
Chapter XXXV, § 7.
(b) Proprietor's Drawings. The fact that drawings by the pro-
k prietor are not expenses of the business and must not be
so construed in preparing single-entry statements is dis-
purchases.
Problem 34
You are asked to prepare an account showing the
profitearned by a concern for a certain period. The
books have been kept by single entry and you gather
from them the following:
Capital $19,;560.00
Cash 2,()00.00
Accounts Receivable 15,600.00
Merchandise 10,400.00
Fixtures 1,650.00
Accounts Payable :i,850.00
Bills Payable 5,000.00
Merchandise used by Proprietor 800.00
98 ILLUSTRATIVE PROBLEMS
Solution to Problem 34
Liabilities:
Accounts Payable $ 3,850.00
Bills Payable 5,000.00 8,850.00
Capital $21,400.00
Deduct:
Cash Drawings by Proprietor $ 1,500.00
Salary Drawn by Proprietor 1,200.00
Merchandise Drawn by Proprietor 800.00 3,500.00
Problem 35
{Wisconsin C. P. A. Examination.)
Solution to Problem 35
Assets
Fixed Assets:
46. Plant Site
47. Plant Buildings
48. Plant, Machinery, and Equipment
68. Workmen's Cottages
40. Office Equipment
24. Good- Will
44. Patent Rights
45. Patterns and Drawings
Investments:
33. Investments (Outside)
64. Sinking Fund Investments
Current Assets:
12. Cash
2. Accounts Receivable
7. Bills Receivable
102 ILLUSTRATIVE PROBLEMS
Current Assets Continued
Inventories:
30. Inventory, Raw Materials
31. Inventory, Goods in Process
32. Inventory, Manufactured Goods
Current Liabilities:
1. Accounts Payable
6. Bills Payable
Accrued Items:
3. Accrued Salaries and Wages
Reserves
53. Reserve for Depreciation of Buildings, Machinery,
and Plant
54. Reserve for Depreciation of Workmen's Cottages
55. Reserve for Doubtful Accounts
56. Reserve for Sinking Fund
Capital
11. Capital Stock
65. Surplus
inventory) I
j
ILLUSTRATIVE PROBLEMS 103
Manufacturing Expenses
39. Non-productive Labor
34. Maintenance of Buildings, Machinery, and Plant
36. Manufacturing Power, Heat, and Light
66. Taxes on Plant and Equipment
25. Insurance
14. Depreciation of Buildings, Machinery, and Plant
37. Miscellaneous Factory Expenses
15. Depreciation of Workmen's Cottages
35. Maintenance of Workmen's Cottages
Trading Accounts:
59. Sales of Manufactured Goods
58. Returns and Allowances on Sales
Selling Expenses:
4. Advertising
62. Salesmen's Salaries
61. Sales Agents'Commissions
63. Salesmen's Expenses
13. Credit Department Expenses
22. Freight and Cartage Outward
38. Miscellaneous Selling Expenses
Capital Income:
17. Discount on Purchases
29. Income from Investments
9. Bond Premium
Problem 36
Goods are supplied to a branch at 120% of cost, or
$24,000, The branch reports an inventory of $15,000;
ILLUSTRATIVE PROBLEMS 107
Solution to Problem 36
Branch A $24,000.00
Purchases $20,000.00
Branch A, Contingent Profit 4,000.00
To record transfer of merchandise of $20,000 at
20% above cost.
Branch A 6,250.00
Branch A, Profit and Loss 6,250.00
For Sales $30,000.00
Less Inventory 15,000.00
$ 6,250.00
Problem 37
Solution to Problem 37
Problem 38
The Good Music Company sells pianos on the instal-
ment plan. On January 2, 1914, Jones purchased a
no ILLUSTRATIVE PROBLEMS
Solution to Problem 38
Jones
Instalment Sales
$375.00
$375.00
Sales
$225.00
Cash
$225.00
Sales $225.00
Piano Purchase $ 75.00
150.00
To credit Profit and Loss with 3/5 f $250.
Cash Payments:
Interest on Notes Payable $ 45.00
Salaries 500.00
Rent 200.00
Sundry Expenses 300.00
Accounts Payable 5,000.00 $6,045.00
Solution to Problem 39
BALANCE SHEET
January 31, 1908
Assets
8
114 ILLUSTRATIVE PROBLEMS
SCHEDULES SHOWING DETERMINATION OF
SUNDRY BALANCE SHEET ITEMS
Cash:
Balance, January 1 $ 500.00
Received 6,600.00 $ 7,100.00
Paid 6,045.00
Notes Receivable:
Balance, January 1 $ 3,000.00
Collected 2,000.00
Accounts Receivable:
Balance, January 1 $5,000.00
Net Charge Sales 7,500.00 $12,500.00
Accounts Payable:
Balance, January 1 $6,000.00
Purchases on Credit 6,000.00 $12,000.00
Payments:
By Cash $5,000.00
By Notes 3,000.00 8,000.00
Deduct — Expenses
Salaries $ 500.00
Rent 200.00
Sundry Expenses 300.00 1,000.00
Problem 40
The firm of Gray and Green has insured the life of
Gray for $50,000, the policy being payable to the firm.
The annual premium is $989.60. The policy does not
have a cash surrender value until the end of the third
year; at that time the surrender value is $2,150.62;
fourth year $2,862.20; year $3,567.25. The pre-
fifth
mium is paid at the beginning of the year.
At the end of the fifth year Gray dies and the policy
is paid to the firm in full.
Solution to Problem 40
First Payment:
Life Insurance Policy — Gray $ 716.87
Life Insurance Expense ili.lS
Cash $ 989.60
Fourth Payment:
Life Insurance Policy — Gray 711.58
Life Insurance Expense 278.02
Cash 989.60
Fifth Payment:
Life Insurance Policy — Gray 705.05
Life Insurance Expense 284.55 /
Cash 989.60
Cash 50,000.00
Life Insurance Policy — Gray 50,000.00
Received cash in settlement from insurance com-
pany.
Problem 41
Required:
(a)
$29,120.00 $29,120.00
$28,275.00 $28,275.00
$28,225.00 $28,225.00
•Solution by D. Himmelblau, C. P. A.
1«2 ILLUSTRATIVE PROBLEMS
(b)
A COMPANY
GENERAL TRIAL BAL
December
'Consists of items indicated by the sales branch house transactions. Other general office or factory
ILLUSTRATIVE PROBLEMS 123
AND BRANCHES
ANCE (BEFORE CLOSING)
31, 1913
1 $ 30,120.00
2 $ 6,020.00
3 6,755.00
4 8,165.00
5 32,090.00
6 $15,640.00 $15,640.00 $15,250.00 $15,250.00 41,270.00
7 2,195.00 1,695.00 2,175.00 1,675.00 1,500.00
8 7,415.00 7,975.00 22,610.00
9 3,820.00 3,650.00 10,920.00
10 17,180.00 16,325.00 52,255.00
11 100.00 250.00 550.00
12 40.00 30.00 120.00
13 10,240.00 10,350.00 32,090.00
14 1,520.00 1,600.00 4,770.00
15 175.00 75.00 450.00
16 125.00 60.00 185.00
17 6,755.00 8,165.00 20,940.00
(c)
A COMPANY
STATEMENT OF PROFIT AND LOSS
For Six Months Ending December 31, 1913
Sales:
On Account $34,525.00 $12,500.00 $11,800.00 $10,225.00
Cash 17,730.00 6,250.00 5,380.00 6,100 00
Cost of Sales:
Inventory, July 1, 1913 . . . $10,920.00 $ 3,450.00 $ 3,820.00 $ 3,650.00
Goods from Factory 32,090.00 11,500.00 10,240.00 10,350.00
Deduct —Expenses
Salaries and Wages $ 4,770.00 $ 1,650.00 $ 1,520.00 $ 1,600.00
Rents and Taxes 450.00 200.00 175.00 75.00
Bad Debts 185.00 125.00 60.00
Problem 42
statements.
The accountant discovers that the following accrued
and prepaid items were ignored by the bookkeeper at
the time of closing the books on June 30, 1919, the date
of the preceding closing, and also at December 31, 1919,
when he had prepared tentative closing entries
126 ILLUSTRATIVE PROBLEMS
June 30, 1919 Dec. 31, 1919
Solution to Problem 42
ADJUSTMENTS
December 31, 1919
Surplus $1,850.00
Profit and Loss $1,850.0C
To charge Surplus with amount of inventory over-
stated as of June 30, 1919.
Surplus 500.00
Interest Accrued on Bonds 500.00
To record the liability for interest on bonds accrued
December 31, 1919, and to charge Surplus with
bond interest accrued as of June 30, 1919.
Problem 43
(I)
Add:
Administration Expense, and Income Deductions:
Legal Expense $ 17.50
Interest Charges 313.00
Insurance 196.23
Telephone 416.06
Total 942.79
Add:
Selling Expense:
Advertising $ 14.00
Commissions 961.01
Total , 975.01
(ID
Total 79,701.94
(HI)
$79,701.94
Deduct— Inventory, May 1, 1914 4,384.61
Selling Expense:
Advertising $ 14.00
Commissions 961.01
Totel 975.01
Total 942.79
(IV)
Problem 44
$7,011.00
Add —Shipments, January 1 to January 10, at cost 516.00
Note: $300 for goods shipped but not billed, as shown by books on
December 31, should be added to accounts receivable.
Problem 45
The books of the X Manufacturing Company were
audited to December 31, 1913, and in making up the
balance sheet and the profit and loss account at that
date the auditors recommended the following adjust-
ments :
i
ILLUSTRATIVE PROBLEMS 133
(a)
BALANCE SHEET
June 30, 1914
Assets
Cash $ 22,143.21
Accounts Receivable 28,250.40
$448,591.74
$448,591.74
(b)
Adjusted amount of profits for half- year to June 30, 1914 $ 71,080.12
(c)
oo o
.'
• (zi
4->
.
.
.OS
o » . .
©^
.
. X
nr
©
SS 'S
X ©
V OO • ""^ •C ©* -! ©' 05
J3
: i2 '- ;
;
;
-o t^ S «5
^~~
eo • F^ • o • * tt
C -I ^
s • •* c e< l>
« : eo
f : d '.<6 ^ f»
aj h •-'5 '^ a>
n Q OS
. . .
—w* --c OS
OS* •
a*
.
» x" -h" ©
o •
a* a»
•
•^ t^ *o
» a»
l^ o ^ © ©o © ©
o
—o o ©© © © © t«
©
«d o d <6 d S © d ^
' •
s< ©
•
1
© © •«
•* © t^
o ©
« X
U a»
u d" ©*
S ---' :
^
>r
,
s^--
_3
•oi^oooooo
.oooooooo t-
< ©
d -1 © © © o o ©
•oeooooooo ^
•
t^aie^Tfoocoo X
o 05
Q
1^ •
* — a» ^' <« d
i-^ ' ©"
H
:3 s
3
co — X „ —
^wO
« o<
c o a* • 5 • * ^ X
d d 00 c6 * d a<
o B
o •
o^o
oo . 05
i»
:
. «3
«5
.
•*
CO
«
«o
i>
X
h
U do'
.
• ^H
.
• A. i
.
«o .-1 oo
v
a oo
—1
<j<
•
•
fM X t* t-
"*
- se- • • • *^
u o — o o Tf a«
© ^
sS "^
:
. <N
:
. •*
;
. q 1-; X
15 sd
(M
•
sri •
d
Ui
d d
»c o *
(^ a»
^ • Tfl • •
t-
/^ a* • p- a* -H <n I* X
io : a» : oo" : d'oo w*
Ci
^H .a) .at .
•-1 00 t~
*u
H as-
a»
«
'f
X H-C •
tjo
c O H
"C
J * •
4) •
4> .2
o 'S
3 ! • J B V 4^
«i •
ffl •
t;
•
. C . 3 w «,'? ;
=3
if
.
-aa
o -
.
CO
C - :
IqW :
4J So
—
5 ' 3 3 »
isa :
03
3
3
°l8
«cJ^
'^
- 03
O
11 t 4) a o
J es I 3 3
di3U <
136 ILLUSTRATIVE PROBLEMS
Problem 46
Following is a transcript of the Surplus account of a
company covering the years 1918 to 1920.
Credits
Debits
Solution to Problem 46
ADJUSTING ENTRIES
(1)
(2)
(3)
(4)
(5)
Inventory 2,000.00
Surplus, 1920 2,000.00
ILLUSTRATIVE PROBLEMS 139
Patents 5,000.00
Surplus, 1920 5,000.00
(7)
(8)
1918 1918
Jan. 15 Dividend $ 20.000,00 Dec. 31 Net Profit. $129,600.00
July 15 Dividend 20.000.00
Dec. 31 Depreciation (1 5,000.00
Adj. of Inventory (2) 1,000.00
Balance 83,600.00
$129,600.00 $129,600.00
1919 1919
Jan. 15 Dividend $ 20,000.00 Jan. 1 Balance $ 83,600.00
July 15 Dividend 20,000.00 Adj. of Inventory (2) 1,000.00
Dec. 31 Good-will 100,000.00 June 30 Adj. of Inventory 2,000.00
Wages (3) 3,000.00 Dec. 31 Net Profit 110,000.00
Balance 54,100.00 Dec. 31 Adj. of Insurance (4) 500.00
$197,100.00 $197,100.00
1920 1920
Jan. 1 Adj. of Insurance (4) $ 500.00 Jan. 1 Balance $54,100.00
Jan. 15 Dividend 20,000.00 Adj. of Wages (S) 3,000.00
July 15 Dividend 20,000.00 Dec. 31 Net Profit 118,000.00
Dec. 31 Suspense (8) 130.00 Adj. of Inventory (5) 2,000.00
Balance 141,770.00 Patent (6) 5,000.00
Insurance (7) 300.00
$182,400.00 $182,400.00
1921
Jan. 1 Balance $141,770.00
140 ILLUSTRATIVE PROBLEMS
Problem 47
A and B were partners, trading under the name of
A, B and Company. June 30, 1920, the following bal-
ances appear on their ledger:
ILLUSTRATIVE PROBLEMS 141
A, Capital $70,000.00
B, Capital 50,000.00
Real Estate 22,000.00
Buildings 20,000.00
Machinery and Tools 44,000.00
Furniture and Fi.xtures 2,000.00
Accounts Receivable . , 50,000.00
Cash 7,000.00
Materials and Merchandise 53,000.00
Accounts Payable 35,000.00
Notes Payable 48,000.00
Notes Receivable 5,000.00
(a) To
prepare closing entries for the books of
A, B and Company.
(b) A statement setting forth the partners'
accounts down to their final closing, be-
ginning with the balances shown by the
books on June 30, 1920.
(c) Opening entries for the Company. X
Solution to Problem 47
(a)
A, AND COMPANY
B
LEDGER BALANCES
June 30, 1920
Cash $ 7,000.00
Accounts Receivable 50,000.00
Notes Receivable 5,000.00
Materials and Merchandise 53,000.00
Real Estate 22,000.00
Buildings 20,000.00
Machinery and Tools 44,000.00
Furniture and Fixtures 2,000.00
Accounts Payable $ 35,000.00
Notes Payable 48,000.00
A, Capital 70,000.00
B, Capital 50,000.00
$203,000.00 $203,000.00
X Company 226,000.00
Cash 7,000.00
Accounts Receivable 48,000.00
Notes Receivable 5,000.00
Materials and Merchandise 53,000.00
Real Estate 17,000.00
Buildings 20,000.00
Machinery and Tools 44,000.00
Furniture and Fixtures 2,000.00
Good- Will 30,000.00
To show sale of assets to the X Company.
A, Capital 7,000.00
Real Estate 5,000.00
Profit and Loss 2,000.00
To show sale to A of real estate of the book
value of $5,000, realizing a profit of $2,000
therefrom.
A, Capital 25,000.00
B, Capital 25,000.00
First Mortgage Bonds 50,000.00
A, Capital 53,000.00
B, Capital 40,000.00
Capital Stock— X Company 93,000.00
To show transfer to the individual partners of
the bonds and capital stock of the Com- X
pany. This closes all partnership accounts.
144 ILLUSTRATIVE PROBLEMS
STATEMENT OF PARTNERS' ACCOUNTS—
Balance, June 30, 1920 $ 70,000.00
Share of Good- Will (50%) 15,000.00
Share of Loss on Bad Debts $ 1,000.00
Share of Profit on Sale of Real Estate 1,000.00
Purchase of Real Estate 7,000.00
First Mortgage Bonds— X Company 25,000.00
Capital Stock— X Company 53,000.00
$ 86,000.00 $ 86,000.00
$ 66,000.00 $ 66,000.00
Cash $ 7,000.00
Accounts Receivable 48,000.00
Xotes Receivable 5,000.00
Materials and Merchandise 53,000.00
Real Estate 17,000.00
Buildings 20,000.00
Machinery and Tools 44,000.00
Furniture and Fixtures 2,000.00
Good- Will 30,000.00
A, B and Company $226,000.00
To bring onto the books the assets acquired
from A, B and Company.
ILLUSTRATIVE PROBLEMS 145
Cash 57,000.00
Capital Stock 57,000.00
Balance of the capital stock (570 shares) has
been sold for cash.
Problem 48
Jan. 1, 1914
July 1, 1914
Jan 1, 1915
July 1, 1915
Jan. 1, 1916
Solution to Problem 48
Total
Coupon Income Amorti- Book
Date Par Value
Receipts at 2% zation Value
at ^y^%
k
(a) Annuities.
Points Illustrated in Problem ^8
Problem 49
You are engaged in auditing the accounts of a cor-
poration for the calendar year 1914 and find on the
ledger an account with the Short Line Railroad bonds,
which was charged on April 1, 1912, with $110,072.42
as the cost of $100,000 first mortgage bonds, payable
January 1, 1917, and bearing interest at 6% payable
January 1 and July 1 of each year.
You observe that no part of the premium has been
charged off, and on investigation you find the following
credits were made to Interest on Investments for the
interest received on the bonds: July 10, 1912, $3,000;
January 8, 1913, $3,000; July 9, 1913, $3,000; January
12, 1914, $3,000; July 10, 1914, $3,000.
You are informed that these bonds were purchased
on a 4%, semiannual basis. Possessed of this informa-
tionyou are required to prepare a schedule showing the
book value of the bonds April 1, 1912, and July 1 and
January 1 of each year; the interest received; the income
and the amortization for each period on the basis on
which the bonds were purchased; the conventional
method being used for the initial fractional period, so
as to furnish necessary information for making future
entries as toincome and amortization. From this data
you are to submit the necessary journal entries to adjust
the accounts to December 31, 1914*
(Ohio C. P. A. Examination.)
ILLUSTRATIVE PROBLEMS 149
The schedule
of amortization which follows has been
setup by the reverse of the usual procedure, that is by-
working backwards, and obtaining each value from the
next later by addition and division.
$103,000.00
$103,980.39
1912 $108,982.59
add simple interest at 4% for three months 1,089.83
•SoluUon by W. P. Kohr, C. P. A.
150 ILLUSTRATIVE PROBLEMS
SCHEDULE OF AMORTIZATION
6% Bonds of the Short Line Railroad Company payable
January 1, Purchased April 1, 1912, at
1917.
$108,572.42 and Accrued Interest.
Jl-Jl
Total Net
Amorti- Book Par
Year Date Interest Income
zation Value Value
6% 4%
*In the problem stated herewith, the interest for the intermediate period is, of course, for three
months.
t"The Accountancy of Investment," by Sprague and Perrine, page 96.
ILLUSTRATIVE PROBLEMS 151
$2,970.97
Problem 50
$298,750.00 $298,750.00
a
c
o
n
O 00
0)
u — 2 ">
5 -g
,£) » <1
t-3
o H H
<N
Cu n
^
c« s
» ><
s
a «
o o
«5
o
en
O
on
o
o
«
^M a*
»
(S«
^N
5 2
o .5
tj -o -5 .S
•cow
ILLUSTRATIVE PROBLEMS 155
fa 52
4; ^^
u •>
O >-•
a ,_c «*- 80
5 >> a
C3
o»
'V ?; **
4j
"O a 'E
<!
CS
3
1
3
OS^
o 3 3 g S
"
e 3 o e
O
•15 y
2^ 3
3 (J
w *^ Q -5
4J
3
O X c8
c 3
03 3 "a.
I I
1 S!
"a
c3
« 1-3
«5 C5
CO 00
«0 50
T5 ^ "c
3
OS
— "o W
"o c 3
3 D J
S «r
^ 3
c
s
•a eS s.
3 X
> to
3
c 3
CO « 3
3 03
c
CS
Q & e 6 a
< 3
-a
5
3 5 ^
to
V V 2 c!
) 3
9 tf u *« !=) n
a
156 ILLUSTRATIVE PROBLEMS
THE ABC ESTATE COMPANY
Exhibit B
STATEMENT OF INCOME AND EXPENDITURES
For the Year Ending December 31, 1913
Income:
Rents $26,400.00
Expenditures:
Agents' Fees and Expenses $2,850.00
General OflSce Expenses 1,050.00
Depreciation of Buildings 6,000.00
Insurance Expired 150.00
Provision for Bad Debts 15.00
Secretary's Salary and Commission 2,380.00
Loss on Bonds Purchased 20.00 12,465.00
from Operations
Profit $13,935.00
—
Deduct Interest on Bonds 5,685.00
Property 6,000.00
Reserve for Depreciation of Buildings 6,000.00
1912 provision credited direct to Property.
Comments on Problem 50
Surplus $135.00
Bonds Purchased $135.00
'^
Loss on bonds canceled during 1912.
Problem 51
{Massachusetts C. P. A. Examination.)
Solution to Problem 51
(a)
(b)
January 1, 1918
Underwriters $450,000.00
Bond Discount 50,000.00
Bonds Authorized $500,000.00
Corporation authorizes an issue of $500,000 5%
bonds in denominations of $500 and $1,000,
interest payable January 1 of each year.
Entire issue of bonds was sold to under-
writers at 90.
February 1, 1918
Cash 475.000.00
Bonds Authorized 500,000.00
Bonds 500,000.00
Underwriters 475,000.00
The company issued the bonds for the under-
writers and received cash in payment.
Underwriters 25,000.00
Cash 25,000.00
Payment of underwriters' profit on issue at
95.
January 1, 1919
January, 1919
Bonds 10,000.00
Bond Discount 300.00
Bond Sinking Fund 9,700.00
The company purchased $10,000 of its bonds in
the open market at 97 and retired and can-
celed them. Interest accrued on the bonds
purchased must be ignored as the exact date
of purchase is not given.
January 1, 1919
Bonds
Interest on 1,000.00
Bond Discount 1,000.00
To remove from Bond Discount the entire
amount of discount applicable to the $10,000
bonds retired.
January 1, 1920
Bond Interest . 24,500.00
Bond Sinking Fund 24,500.00
To show payment of January, 1920, interest
on bonds outstanding on that date.
January 1, 1920
Bonds 11,000.00
Bond Discount 220.00
Bond Sinking Fund 10,780.00
To show purchase of 11 $1,000 bonds from the
bond sinking fund at 98.
January 1, 1920
on Bonds
Interest 1,100.00
Bond Discount 1,100.00
To remove from Bond Discount the entire
amount of discount applicable to the $11,000
bonds retired on above date.
164 ILLUSTRATIVE PROBLEMS
(c)
TRIAL BALANCE
January, 1920
Cash $380,000.00
Bond Sinking Fund 20.00
Bond Discount Unextinguished 47,380.00
Bond Interest 51,600.00
Bonds $479,000.00
$479,000.00 $479,000.00
Comments on Problem 51
The bond discount should be amortized but no
basis is given by which this may be done inasmuch as
the Hfe of the bonds is not given. From the fact that
7% of the bond issue, or $35,000, is set aside annually
into the sinking fund, $25,000 of which was used the
first year in paying interest, leaving $10,000 to redeem
§12.
(d) Bond Discount. The nature of bond discount and the ac-
counting means for distributing it over the life of the bond
issue are discussed in Volume IV, Chapter X, § 10.
(e) Underwriters' Profit. This case illustrates the profit made
by underwriters to whom bonds were issued at 90 and for
whom they were sold at 95.
Problem 52
Solution to Problem 52
Problem 53
Solution to Problem 53
Problem 54
{Wisconsin C. P. A. Examination.)
Solution to Problem 54
Total Ex-
Scrap Deprecia- Times Dollar
Value Life penditure
Value tion Replaced Years
in 50 Yrs.
Problem 55
Solution to Problem 55
The Prosperous Company, a corporation incorporated
under the laws of the State of New York, with an author-
ized capital stock of $250,000 preferred stock, and $250,000
common stock, shares being of the par value of $100 each.
Common Stock
Subscribers to $ 500.00
Common Stock Subscriptions $ 500.00
To record subscriptions to five shares of com-
mon stock, one share each, by the five in-
corporators.
$599,500.00
Cash 500.00
Subscribers to Common Stock 500.00
To record payment of subscriptions.
Assets
Cash $ 12,500.00
Accounts Receivable $33,000.00
Less— Reserve 5,000.00 28,000.00
Inventories 100,000.00
Land and Buildings 75,000.00
Plant and Machinery 200,000.00
Tools, Equipment, and Fixtures 50,000.00
Good- Will 134,500.00
$600,000.00
Liabilities
$600,000.00
ILLUSTRATIVE PROBLEMS 173
Problem 56
Solution to Problem 56
Cash $ 300.0
Capital Stock 300.00
To record the sale for cash of a share of stock
each to X, Y, and Z.
A, Vendor 80,000.00
Sundry Liabilities 80,000.00
To record the liabilities assumed.
A, Vendor 70,000.00
Good- Will 29,700.00
Capital Stock 99,700.00
To show the issue to A of the entire capital
stock of the company with the exception of
the three shares sold for cash. As the
amount of stock issued to A is $29,700 in
excess of the net worth of the property ac-
quired from him, the excess represents the
valuation expressed in stock which was
placed upon the good-will of the business
acquired from him.
Cash 2,400.00
Treasury Stock 2,400.00
To record the sale of 50 shares of treasury
stock at 48.
Cash 2,600.00
Treasury Stock 2.600.00
To record the sale of the remaining 50 shares of
treasury stock at 52.
ILLUSTRATIVE PROBLEMS 175
(a) Treasury Stock. The nature of treasury stock and the pur-
pose for which it is used are described in Volume I, Chap-
ter XXXIII, § 9, and further elaborated in Volume IV,
Chapter XV, § 3.
(b) Surplus. When the donated treasury stock is sold, a clear
increase in surplus results. Some accountants prefer to
show surplus from this source in a special account ear-
marked by some title which will indicate its nature. There
seems to be no reason for this because such surplus is
av^ailable for distribution as dividends. If the corpora-
tion as a matter of financial policy decides not to dis-
tribute then a reserve should be created, but the original
it,
Solution to Problem 57
Patents 100,000.00
Unissued Preferred Stock 50,000.00
Unissued Common Stock 50,000.00
Patents are acquired of the patentee, preferred
stock to the amount of $50,000 and common
stock of the same amount being issued in
payment thereof.
BALANCE SHEET
As at
Assets
Patents $100,000.00
Issued .
$ 50,000.00
Less — Held in Treasury 25,000.00
Outstanding $ 25,000.00
Issued $ 50,000.00
Less — Held in Treasury 25,000.00
Outstanding 25,000.00
Surplus from Treasury Stock Donated 50,000.00
$100,000.00
Problem 58
JOURNAL ENTRIES
Coupon Advertising (at 75 cents per 100) $
Estimated Lapses (at 25 cents per 100)
Unredeemed Coupons (at $1 per 100) $
Coupons contained in hundred boxes sold
during month.
Premiums
Accounts Payable (or Cash)
Premiums purchased during the month (at $1
each).
Unredeemed Coupons
Premiums ,
Comments on Problem 58
"Coupon Advertising" covers the expense for the
month.
Balance of " Premiums " is value of premiums on
hand.
Balance of " Unredeemed Coupons " is gross liabil-
ity.
Problem 59
Solution to Problem 59
Surplus $1,640.00
Wages and Salaries $1,640.00
The above entry is to debit Surplus account with
wages paid in 1920 but representing expenses of
the preceding year.
Insurance 360.00
Surplus 360.00
To credit Surplus account with insurance unex-
pired as at December 31, 1919, but incorrectly
considered as an expense of 1919.
Surplus 1,200.00
Interest on Mortgage 1,200.00
To debit Surplus account with interest on mortgage
paid in 1920, but representing an expense of the
preceding year.
Surplus 8,000.00
Merchandise Inventory 8,000.00
To credit Inventory account with goods received
in 1919, but which were not recorded as pur-
chases, and hence should not have entered into
the profit computation for 1919. Surplus ac-
count should show a consequent reduction, and is
therefore debited.
182 ILLUSTRATIVE PROBLEMS
Merchandise Inventory 1,500.00
Surplus 1,500.00
To adjust the Inventory account as at the beginning
of this year, it having been figured at too small an
amount by $1,500. A consequent increase in
profits for 1919 results, hence Surplus account
is credited.
Surplus 6,500.00
Reserve for Depreciation on Real Estate 6,500.00
Estimated depreciation on real estate was not com-
puted in 1919. The above entry reduces the
Surplus account on the basis of the depreciation
that should have been considered as an expense of
1919.
Problem 60
A is the proprietor of a business. His books were
December 31, 1919, at which time his capital
closed as of
account showed a credit balance of $18,000. He oflFers
to sell B
a one-third interest in the business for $10,000,
which offer is accepted by B. Prepare balance sheet
of A and B.
Solution to Problem 60
$30,000.00 $30,000.00
$18,000.00 $18,000.00
Comments on Problem 60
Cash $10,000.00
B . $10,000.00
To bring on the books the cash invested by B.
$30,000.00 $30,000.00
(a) Good- Will. The nature of good- will and the circumstances
under which it may properly be set up on the books are
discussed in Volume IV, Chapter XIX, §§ 2 and 7.
(b) Capital Accounts. This problem illustrates the function of
the capital account for a partner. (Refer to Volume I,
Chapter XXXI, § 3.
(d) Profit and Loss Distribution. As explained in Volume I,
186 ILLUSTRATIVE PROBLEMS
Chapter XXXI, § 6, partners may agree upon any basis
for the distribution of profits or losses. In that chapter
there are suggested several methods which are in common
use.
Problem 61
Solution to Problem 61
X $5,500.00
Y 9,500.00
Z $15,000.00
X $2,000.00
Y 6,000.00
Z $8,000.00
(si) Good- Will. For a discussion of good- will and all the nec-
essary accounting treatment of it, see Volume IV, Chapter
XIX.
(b) Admission of Partner. The bookkeeping necessary when a
new partner is admitted to a firm is explained in Volume
I, Chapter XXXI, § 3.
Problem 62
Solution to Problem 62
(A, $60,000
Assets, $125,000 B, 40,000
Ic, 25,000
A, Capital
$60,000.00
B, Capital
$40,000.00
C, Capital
I
Cash $25,000.00
Distribution of profit:
A — 48%
B — 32%
C - 20%
As C is to receive }/^ of the profits, he gets 20%.
A and B are to split the remaining 80%, Ys to A
and ~/s to B, as previously.
80% X Yo = 48% to A
80% X % = 32% to B
190 ILLUSTRATIVE PROBLEMS
Problem 63
Sales 55,188.00
Purchases 27,804.00
Wages and Salaries 4,600.00
General Expenses 1,560.00
Partners' Drawing Accounts:
C, debit balance 5,500.00
" "
D, 5,500.00
" "
E, 5,500.00
Cash in Bank and on Hand 3,974.00
Solution to Problem 63
C, D, AND E PARTNERSHIP
ADJUSTING ENTRIES
June 30, 1916
C, D, AND E PARTNERSHIP
PROFIT AND LOSS STATEMENT
For the Year Ended June 30, 1916
Deduct — Expenses :
Distribution of Profits:
C— Va $ 6,641.67
D— Vs 6,641.67
E— Vs 6,641.66 $19,925.00
C, D, AND E PARTNERSHIP
BALANCE SHEET
June 30, 1916
Assets
Plant and Equipment $14,840.00
Less— Reserve for Depreciation 1,484.00 $13,356.00
ILLUSTRATIVE PROBLEMS 193
Cash 3,974.00
Inventory $ 7,600.00
Less— Reserve for Depreciation .... 5,388.00 2,212.00
Liabilities
Capital:
C— Investment $ 8,000.00
Less — Drawings 5,500.00
$ 2,500.00
Plus Profit for period 6,641.67 $ 9,141.67
D— Investment $ 8,000.00
Less — Drawings 5,500.00
$ 2,500.00
Plus Profit for period 6,641.67 9,141.67
E— Investment $ 8,000.00
Less — Drawings 5,500.00
$ 2,500.00
Plus Profit for period 6,641.66 9,141.66 27,425.00
Chapter XXXII, § 5.
Problem 64
Solution to Problem 64
Cash 35,000.00
Fire Loss 35,000.00
(Explanation.)
Surplus 13,000.00
Fire Loss 13,000.00
To record fire loss not compensated for by insurance.
Merchandise Debits
Merchandise Credits
appraised at $6,000.
Make all entries necessary to adjust matters, includ-
ing the separation of the single Merchandise account
into separate accounts; prepare a statement showing
details of the loss, a statement showing gross profit on
sales to time of fire, and entries to close all accounts given.
ILLUSTRATIVE PROBLEMS 197
Solution to Problem 65
Real Estate:
Cost $12,125.00
Less:
Insurance $5,000.00
Value of Land 6,000.00 11,000.00
Sales $10,059.46
Cost of Sales:
Inventory, January 1 $3,372.55
—
Purchases Gross $14,152.39
Freight and Cartage 377.32 $14,529.71
Less:
Cost of Goods Destroyed $ 8,920.38
Cost of Consigned Goods 2,745.50 "
11.665.88 2,863.83 6,236.38
Trading 3,372.55
Inventory 3,372.55
To close the inventory as of January 1 into
Trading.
Trading 377.32
Freight and Cartage 377.32
To close freight and cartage into Trading.
Trading 2,863.83
Purchases 2,863.83
To close the net cost of goods sold during the
period into Trading.
Trading 3,823.08
Profit and Loss 3,823.08
To close the gross profit on sales into the Profit
and Loss account.
ILLUSTRATIVE PROBLEMS 199
Surplus 5,753.28
Fire Loss 5,753.28
To close the net fire loss into Surplus.
(a) Fire Loss. See Volume IV, Chapter XX, § 8, for a general
description of the bookkeeping necessary to record fire
losses.
(b) Merchandise Account. In Volume I, Chapter VIII, § 3,
the Merchandise account which formerly was in common
use is described. The inconvenience of such an account
and the necessity for analyzing it can be noted in this
problem.
(c) Consignments. When goods are shipped on consignment,
no sale has taken place and accordingly the credit should
not be to a Sales account but may properly be to the
Purchases or Inventory account as shown in this problem.
The reason for this is that profit must not be anticipated
and the credit for the sale must be deferred. (See Volume
IV, Chapter III.)
(d) Freight Inward. For a discussion of the adding of freight
inward to the cost of purchases, see Volume I, Chapter
IX. § 8.
Problem 66
Equipment $ 75,000.00
Merchandise 100,000.00
Accounts Receivable 80,000.00
$255,000.00
Equipment $ 15,000.00
Merchandise 25,000.00
Book Debts 50,000.00
$290,000.00 $290,000.00
(a) Fire Loss. The most practicable way of recording the facts
involved in fire losses is discussed in Volume IV, Chapter
XX, § 8.
(b) Surplus. The balance of the Fire Loss account is closed
into Surplus because of the unusual nature of the transac-
tions involved. For other adjustments of surplus of a
similar nature, see Volume IV, Chapter VII, § 4.
(c) Unrealized Profit. As suggested in the solution of this prob-
lem, when any doubt exists as to the ultimate collection
and realization of asset values, the credit for such values
should be deferred until the realization actually occurs.
Accordingly, in this problem the balance of the Fire Loss
Adjustment account was carried as a reserve until the
salvage had been realized upon. This method of deferring
credits is explained in Volume IV, Chapters III and VIII.
Problem 67
CORPORATION A
Assets Liabilities
Plant and Machinery . $ 35,000.00 Common Stock $ 50,000.00
Raw Materia! 6,500.00 Preferred Stock 42,500.00
Work in Process 9,200.00 Preferred Stock Scrip . 7,500.00
Finished Product . . . 16,700.00 Surplus 5,400 00
Accounts Receivable . 33,500.00 Accounts Payable .... 14,200.00
Bills Receivable 14,500.00
Deferred Charges 1,200.00
Cash 3,000.00
$119,600.00 $119,600.00
CORPORATION B
Assets Liabilities
Plant and Machinery . $ 51,000.00 Capital Stock $100,000.00
Inventories 32,000.00 .\ccounts Payable .... 31,610.00
Accounts Receivable . 47,500.00 Reserve for Bad Debts 1,940.00
Cash 1,000.00
Deficit 2,050.00
$133,550.00 $133,550.00
Solution to Problem 67
Cash $ 500.00
Common Stock $ 500.00
Issue of 5 shares of common stock for cash.
Cash 3,000.00
Plant and Sundry Assets 222,000.00
Accounts Payable 14,200.00
Corporation A, Vendor 210,800.00
Purchase of plant and assets of Corporation A
in accordance with contract approved by
board of directors.
Cash 1,000.00
Plant and Sundry Assets 228,450.00
Accounts Payable 31,610.00
Reserve for Bad Debts 1,940.00
Corporation B, Vendor 195,900.00
Purchase of plant and assets of Corporation B
in accordance with contract approved by
board of directors.
CORPORATION C
BALANCE SHEET
Assets
Capital Assets:
Good- Will $203,350.00
Plant and Machinery 86,000.00
Total $289,350.00
Current Assets:
Inventories $ 64,400.00
Accounts Receivable $81,000.00
Less— Reserve for Bad Debts . . 1,940.00 79,060.00
Total 162,460.00
Total $453,010.00
Total $407,200.00
Current Liabilities:
Accounts Payable 45,810.00
Total $453,010.00
206 ILLUSTRATIVE PROBLEMS
XIX, § 4.
(e) Reserve for Bad Debts. The reserve for bad debts is de-
ducted on the balance sheet from the gross amount of
accounts receivable. This is in accordance with the prac-
tice which is recommended in Volume I, Chapter XXIX,
§11.
(f) Purchase of Deferred Charges. In this example the new cor-
poration acquired, among the assets purchased, deferred
charges previously carried by Corporation A. Unless the
deferred charges represented an organization expense of
Corporation A, they may be presumed, from the state-
ment of the problem, to be a current item such as pre-
paid insurance or the like. In that event, it is properly
chargeable as an asset in the accounts of the new corpora-
tion.
ILLUSTRATIVE PROBLEMS 207
Problem 68
In January 15, 1901, Howard Robinson and four
others acquired a tract of 600 acres at a cost of $20,000.
On March 1, 1901, they incorporate the Nob Hill
Realty Company for the purpose of acquiring, sudivid-
ing, and selling this tract for residence purposes.
The par value of the stock is $100 per share, the
capital $300,000, ofwhich $120,000 is issued for the
land purchased, and the balance $180,000 is paid for
in cash. The directors engage a landscape architect to
lay out the tract, a special feature of which is to be a
beautiful park, together with tree-lined boulevards and
driveways. In accord with the architects' advice the
directors defer marketing any portion of the property
until the year 1911.
Owing to errors in early development work the com-
pany is compelled to borrow $50,000 at 6% on March
1, 1910. The loan is secured by a mortgage on the en-
tire property, with the customary release clause for
individual lots upon payment of $25,000 of the loan,
and $1,000 on each lot for which release is demanded.
Sales of lots are made beginning March 1, 1911.
The sale contract provides that the company will
maintain the park and driveways in perpetuity and to
insure this a fund will be created for the permanent
maintenance and care of the park and driveways, the
estimated annual expense of which is $6,000. It is
agreed with purchasers of lots that one-third of all
cash received from sales shall be invested in sound
bonds yielding 4% net, until $150,000 has been so in-
vested. It is further agreed that upon the sale of all
208 ILLUSTRATIVE PROBLEMS
TRIAL BALANCES
Mar. 1, 1911 Feb. 28, 1917
Debits:
Cash on Hand $ 16.000.00 $ 252,000.00
Bonds 100,000.00
Real Estate 120,000.00 120,000.00
Improvements 160,000.00 160,000.00
Improvements Replaced 60,000.00 60.000.00
General Expense 24,000.00 84,000.00
ILLUSTRATIVE PROBLEMS 209
$380,000.00 $1,242,000.00
Credits
Capital Stock $300,000.00 $ 300,000.00
Sale of Lots 900,000.00
Bond Interest 12,000.00
Interest (on call loans) 30,000.00 30,000.00
Mortgage 50,000.00
$380,000.00 $1,242,000.00
Solution to Problem 68
March 1, 1901
Cash 50,000.00
Mortgage 50,000.00
A loan of $50,000 is secured by a mortgage on
the entire property. Interest on mortgage,
6%.
14
210 ILLUSTRATIVE PROBLEMS
Improvements 160,000.00
Improvements Replaced 60,000.00
Cash 220,000.00
To show expenditure of cash for improvement
of real estate.
Cash 30,000.00
Interest 000.00
$30,000 interest received on call loans.
Cash $900,000.00
Sales of Lots $900,000.00
To record the sale of .SCO lots at $2,500 each.
Bonds 100,000.00
Cash 100,000.00
To record purchase of bonds for permanent
maintenance of park.
Cash 12,000.00
Bond Interest 12,000.00
To show income from bonds purchased for
investment.
Mortgage 50,000.00
Cash 50,000.00
To show the taking up of the mortgage placed
on the property.
—
Park Fund Expense and Income 36,000.00
Park and Driveway Maintenance 36,000.00
To close park and driveway maintenance ex-
penses into the Expense and Income account.
Note: The excess of expenses over income with respect to the mainte-
nance of the property must be considered a reduction of the income from sales
until such time as the permanent fund is suflSciently large to carry the burden
of maintaining the property.
Surplus 150,000.00
Reserve for Park Maintenance 150,000.00
To establish a reserve account against the
amount of $150,000 which is to be turned
over to the Nob Hill trustees. (Note:
This amount must be taken out of Surplus
in order to prevent its use for dividend dis-
tribution.)
Surplus 365,400.00
Dividends in Liquidation 64,600.00
Dividends Paid 430,000.00
To close Dividends Paid into Surplus and into
Dividends in Liquidation.
Assets
Real Estate (40 salable lots at $835) $ 33,400.00
Cash 202,000.00
Park Fund:
Bonds $100,000.00
Cash 50,000.00 150,000.00
Capital
Capital Stock $300,000.00
Less — Dividends in Liquidation 64,600.00 $235,400.00
Problem 69
Prepare a statement of the operations of a railroad
company (using your own figures) and show the profit
and loss account after providing for an amortization of
5% of the gross earnings; also state the assets and lia-
bilities of the company.
(Pennsylvania C. P. A. Examination.)
Other Income:
Income from Lease of Road $ 218,545.90
Hire of Equipment— Credit Balance 393,218.19
Joint Facility — Rent
Income 259,555.56
Miscellaneous Rent Income 28,450.79
Net Profit from Miscellaneous Physical
Property 81,962.58
Dividend Income 915,313.89
Income from Funded Securities 224,780.40
Income from Unfunded Securities and Ac-
counts 691,149.82
Debits
Dividend Appropriations of Surplus:
Cash Dividend, 33^%, payable February
10, 1914 $ 2,520,000.00
Cash Dividend, 3j^%, payable August
10, 1914 2,520,000.00
$ 5,040,000.00
Debt Discount Extinguished
through Surplus $273,687.50
Less — Premium Realized on
Bonds sold during year . . 175,000.00 98,687.50
Miscellaneous Debits:
Amount appropriated to Reserve for
Additions to Property — being 5% of
Gross Earnings from Rail Operations,
viz., 5% of $59,682,777.77 2,984,138.89
$241,818,427.99
Reserve for Accrued Depreciation — Credit:
Way and s>tiu< lures, etc $ 9,974,264.55
Equipment 17,439,335.40 27,413,599.95 $214,404,628.04
Seccrities:
Proprietary, Affiliated, and Controlled Companies:
Pledged:
Stocks $ 250,728.48
Funded Debt 1,200,000.00 $ 1,450,728.48
Otheb Investments:
Advances and Controlled Companies
to Proprietary, Affiliated
for Construction, Equipment and Betterments $ 18,012,919.^8
Miscellaneous Investments:
Physical Property $ 3,220,078.08
Securities— Pledged 14.662,472.37
Securities — Unpledged:
Stocks $4,288,753.08
Miscellaneous 16,442.44 4,305,195.52 22,187.745.97 41,100,665.25
Working Assets:
Cash ... $ 13,815.564.10
Securities Issued or Assumed — Held in Treasury — Funded Debt. 10,644,339.94
Marketable 55ecurities 6,786,768.75
Loans and Bills Receivable 214.234.85
Traffic and Car Service Balances Due from Other Companies. . . 496,297.44
Net Balance Due from .\gents and Conductors '
988.523.58
Miscellaneous Accounts Receivable 3,174.412.88
Materials and Supplies 7,086,383.22 43,208,524.76
$313,300,279.45
Liabilities
Stock:
Capital Stock:
Common Stock:
Full Shares Outstanding $ 71,917,200.00
Fractional Shares Outstanding 720.00
Original Stock and Subsequent Stock Dividends Unissued . 82,080.00
$ 72,000,000.00
Premium Realized on Capital Stock 12,116.76 9 72,012,116.76
Working Liabilities:
Trafficand Car Service Balances Due to Other Companies. 238,551.54
Audited Vouchers and Wages Unpaid 2,782,527.38
Miscellaneous .\ccounts Payable 541,189.92
Matured Interest, Dividends, and Rents Unpaid 1,257,811.17
Matured Mortgage, Bonded, and Secured Debt Unpaid. . . 95,000.00
Other Working liabilities 472,310.28 5.437,390.29
Appropriated Scbplus:
Additions to Property since June 30. 1907, through Income or
Surplus 9 2,984,138.89
Reserves from Income or Surplus:
For Doubtful Accounts 290,454.08 3.274.692.97
$313,300,279.45
220 ILLUSTRATIVE PROBLEMS
Problem 70
From the trial balance of the Wonder Machine
Shoe Company prepare a balance sheet and statement
with sections showing manufacturing costs, trading
results, and profit and loss.
Reserve for the depreciation of machinery, 10%; of
tools, 10%; of lasts and patterns, 20%. Reserve for
loss from bad debts an amount that, when added to the
reserve for that purpose already in force, will make the
sum 1% of the book accounts.
TRIAL BALANCE
December 31, 1919
$695,225.88 $695,225.88
INVENTORIES
December 31, 1919
Solution to Problem 70
Interest 65.00
Interest on Notes Payable Accrued 65.00
To record interest due on notes outstanding.
224 ILLUSTRATIVE PROBLEMS
Assets
Fixed Assets:
Real Estate $81,035.00
Machinery and Equipment $57,750.00
Less — Reserve for Depreciation . 5,775.00 51,975.00
Tools $ 5,259.00
Less — Reserve for Depreciation . 525.90 4,733.10
Current Assets:
Accounts Receivable $62,316.50
—
Less Reserve for Loss on Bad
Debts 623.17 $61,693.33
Inventories:
Raw Materials $ 5,397.24
Goods in Process 18,493.12
Finished Goods 8,898.61
Factory Supplies 820.20
Fuel 1,592.17 35,201.34
Accrued Items:
Interest on Notes Receivable Accrued 37.00
Current Liabilities:
Accounts Payable $18,580.70
Bills Payable 6,500.00 25,080.70
Accrued Items:
Interest on Notes Payable Accrued 55.00
Surplus
Balance of Surplus, December 31, 1919 $ 7,329.46
Add — Net Profit for period 55,692.94 63,022.40
—
Deduct Cost of Manufactured Goods Sold:
On Hand, January 1, 1919 $ 15,686.31
Cost of Goods Manufactured
during year (See Exhibit C) . . . 311,049.77 $326,736.08
Deduct:
Selling Expense $ 25,792.65
General Expense 16,123.75 41,916.40
Labor 110,371.84
Manufacturing Expenses
Manufacturing Expenses $ 12,688.13
Indirect Labor 5,193.00
Factory Supplies 7,997.42
Depreciation of Machinery 5,775.00
Depreciation of Tools 525.90
Depreciation of Lasts and Patterns 7,052.00 39,231.45
$329,542.89
Deduct— Goods in Process, December 31, 1919 18,493.12
Cost of Sales
Operating Expenses, Material and Supplies . . . $257,000.00
Plant Expense 12,000.00
Freight on Returned Goods 600.00
Sundry Purchases Finished Goods 10,400.00 280,000.00
Other Income:
Miscellaneous Earnings $ 1,500.00
Profiton Contracts 6,500.00
Discount on Purchases 500.00 8,500.00
$ 28,500.00
Less:
Discount on Sales $ 2,875.00
Rebates and Allowances 1,125.00 4,000.00
Less:
General Expenses $ 5,500.00
Interest 1,500.00 7,000.00
Solution to Problem 71
Wages 122,500.00
Add:
Finished Goods Inventory, July 1, 1907 $ 45,000.00
Sundry Purchases of Finished Goods 10,400.00 55,400.00
$324,400.00
Less — Inventory Finished Goods, June 30, 1908 60,000.00
Add:
Miscellaneous Earnings $ 1,500.00
Profit on Contracts 6.500.00 8,000.00
Deduct:
General Expenses .'
$ 5,500.00
Freight on Returned Goods 600.00 6,100.00
Problem 72
Solution to Problem 72
Sales $540,000.00
—
Deduct Other Charges:
Bad Debts 8,000.00
Problem 73
Land $ 10,000.00
Buildings 75.000.00
Machinery 119,138.73
Tenements 1,670.66
Finished Goods Inventory, January 1, 1916 66,984.43
Stock in Process Inventory, January 1, 1916 .... 57,042.38
Yarn . 259,882.12
Cash ,".T7.T7rr.T. 12,769.19
Petty Cash 106.39
Accounts Receivable 46,085.68
Mortgage Receivable 875.00
Labor 25,979.27
Supplies 2.974.31
2S4 ILLUSTRATIVE PROBLEMS
Repairs 956.63
Oils 50.84
Coal 1,443.20
Starch 1,390.00
Water 122.65
Finishing 15,381.54
Brokerage 660.50
Commission 4,580.67
Discounts Allowed 1,246.84
Insurance 679.92
Taxes 1,502.81
General Expense 389.39
Freight and Express 974.34
Telephone and Telegraph 68.72
Traveling Expense 274.85
Interest Paid 409.80
Discount on Notes Payable 1,408.00
Profit and Loss 20,694.80
Dividends 3,375.00
Capital Stock, Preferred 6% Cumulative $100,000.00
Capital Stock, Common 263,800.00
Accounts Payable 40,864.56
Notes Payable 187,500.00
Cloth Sales 137,818.07
Waste Sales 922.94
Tenement Rents Received 339.50
Discounts Taken 2,873.59
$734,118.66 $734,118.66
Solution to Problem 73
TEXTILE MILL
ADJUSTING ENTRIES
June 30, 1916
Dividends 6,000.00
Preferred Dividends Payable 6,000.00
Cumulative dividends on preferred stock unpaid
for the year ended June 30, 1916.
Labor 2,051.05
Wages Accrued 2,051.05
Accrued wages to date.
TEXTILE MILL
STATEMENT SHOWING COST OF GOODS
MANUFACTURED
For the Six Months Ended June 30, 1916
$210,267.53
Deduct— Stock in Process, June 30, 1916 71,242.39
Manufacturing Expenses
Finishing $ 15,381.54
Oils 50.84
Coal $ 1,443.20
Less— Inventory 1,000.00 443.20
Starch $ 1,390.00
Less— Inventory 900.00 490.00
Water $ 122.65
ft Less— Rates Prepaid 100.00 22.66
^ Supplies $ 2,974.31
^ Less— Inventory 1,150.00 1,824.31
Repairs 956.63
Insurance $ 679.92
Less— Unexpired 600.00 79.92
238 ILLUSTRATIVE PROBLEMS
Taxes $ 1,502.81
Less— Prepaid 402.26 1,100.65
Depreciation
On Machinery $ 2,978.47
On Tenements 25.00
On Mill Buildings 750.00 3,753.53 24,103.17
Deduct
Waste
Sales of $ 922.94
Tenement Rents Received 339.50 1,262.44
TEXTILE MILL
PROFIT AND LOSS STATEMENT
For the Six Months Ended June 30, 1916
Deduct —
Cost of Goods Sold
—
Inventory Finished Goods, Jan-
uary 1, 1916 $ 66,984.43
Add — Cost of Goods Manufac-
tured 161,865.87 $228,850.30
Deduct Expenses:
Traveling Expenses $ 274.85
Commissions 4,580.67
Telephone and Telegraph 68.72
Brokerage 660.50
General Expense 389.39 5,974.13
Add —
Other Income:
Discounts Taken 2,940.88
TEXTILE MILL
BALANCE SHEET
June 30, 1916
Assets
Fixed Assets:
Land $ 10,000.00
Buildings $ 75,000.00
Less — Reserve for Depreciation 1,500.00 73,500.00
Machinery $119,138.73
Less — Reserve for Depreciation 5,956.94 113,181.79
Tenements $ 1,670.66
Less — Reserve for Depreciation 50.12 1,620.54
Current Assets:
Cash in Bank $ 12,769.19
Petty Cash 106.39 $ 12,875.58
Inventories:
Finished Goods $104,190.24
Stock in Process 71,242.39
Yarn 135,661.63
240 ILLUSTRATIVE PROBLEMS
Coal 1,000.00
Starch 900.00
Supplies 1,150.00
Prepaid Items:
Interest Prepaid on Notes Payable ... $ 211.11
600.00
Prepaid Taxes 402.26
100.00
Current Liabilities:
Accounts Payable $ 40,864.56
Notes Payable 187,500.00
Interest Accrued on Notes Payable 389.41
Wages Accrued 2,051.05
Estimated Discounts to be Allowed 460.86
Preferred Dividends Payable 6,000.00
Capital
Capital Stock, Preferred $100,000.00
Capital Stock, Common 263,800.00 $363,800.00
$ 33,473.33
Deduct— Profits for the period . 6,720.96
Problem 74
GEORGE W. DUNN
TRIAL BALANCE
December 31, 1920
Cash $ 5,627.00
Accounts Receivable 229,296.00
Notes Receivable 22,600.00
Inventory, December 31, 1919 (cost) 215,275.00
Accrued Interest on Notes Receivable 654.00
Real Estate (book value) 161,540.00
Store Fixtures (book value) 19,416.00
OflBce Furniture and Fixtures (book value) 2,760.00
Prepaid Interest on Discounted Notes 1,350.00
Catalogues and Advertising Matter on Hand . 956.00
Prepaid Taxes 897.00
Prepaid Insurance 175.00
Accounts Payable $ 89,264.00
Notes Payable 50,000.00
Mortgage Payable 70,000.00
Accrued Interest on Notes Payable 560.00
Accrued Interest on Mortgage Payable 700.00
George W. Dunn, Capital 370,000.00
George W. Dunn, Drawings 15,677.00
Sales 802,071.00
Sales Returns and Allowances 17,200.00
Purchases 599,025.00
Purchase Returns and Allowances 8,672.00
Freight and Hauling Inward 4,130.00
ILLUSTRATIVE PROBLEMS 243
Advertising 19,607.00
Store Clerks' Salaries 20,460.00
Traveling Salesmen's Salaries 18,643.00
Traveling Expenses 13,721.00
Store Supplies Used 1,416.00
Freight and Hauling Outward 2,160.00
OflSce Clerks' Salaries 7.482.00
Office Expenses 1,786.00
Maintenance of Real Estate 14,682.00
Income from Rental of Upper Floors 11,627.00
Interest on Notes Receivable 1,287.00
Interest on Bank Balances 96.00
Cash Discounts on Purchases 4,893.00
Interest on Notes Payable 5,740.00
Interest on Mortgage Payable 4,200.00
Cash Discounts on Sales 2,695.00
$1,409,170.00 $1,409,170.00
Required
(a) Profit and loss statement
(b) Balance sheet
(c) Closing entries
(a)
GEORGE W. DUNN
PROFIT AND LOSS STATEMENT
One Year Ending December 31, 1920
Administrative Expenses:
Office Clerks' Salaries $ 7,482.00
OfficeExpenses 1.786.00
Maintenance of
Real Estate ... $ 14,682.00
Less — Income
from Rental
of Upper
Floors 11,627.00 3,055.00 12,323.00 88,330.00
(b)
GEORGE W. DUNN
BALANCE SHEET
December 31, 1920
Assets
Current Assets:
Cash $ 5,627.00
Accounts Receivable 229,296.00
Notes Receivable 22,600.00
Merchandise on Hand (cost) 176,482.00
Accrued Items:
Interest on Notes Receivable 654.00
Fixed Assets:
Real Estate (book value) $161,540.00
Store Fixtures (book value) 19,416.00
OflSce Furniture and Fixtures (book value) .... 2,760.00
$621,753.00
246 ILLUSTRATIVE PROBLEMS
Liabilities and Capital
Current Liabilities:
Accounts Payable $ 89,264.00
Notes Payable 50,000.00
Accrued Items:
Interest on Notes Payable $ 560.00
Interest on Mortgage Payable . . 700.00 1,260.00
Fixed Liabilities:
Mortgage Payable 70,000.00
$621,753.00
(c)
GEORGE W. DUNN
CLOSING ENTRIES
December 31, 1920
Sales $ 17,200.00
Sales Returns and Allowances $ 17,200.00
To close the sales returns and allowances for
the year into the Sales account.
Purchases 4.130, 00
Freight and Hauling Inward 4,130.00
To close the cost of freight and hauling inward
on merchandise purchases for the year into
the Purchases account.
ILLUSTRATIVE PROBLEMS 247
Purchases 215,275.00
Inventory 215,275.00
Cost of goods on hand December 31, 1919.
Inventory 176,482.00
Purchases 176,482.00
Cost of goods on hand December 31, 1920.
Sales 633,276.00
Purchases 633,276.00
To close the cost of goods sold during the year
into the Sales account:
Inventory, December 31, 1919 .... $215,275.00
Gross Purchases $599,025.00
Less —
Returns and
Allowances 8,672.00
$809,758.00
Less— Inventory, December 31, 1920 176,482.00
$633,276.00
Sales 151,595.00
Profit and Loss 151,595.00
To close the gross profit on sales for the year
into Profit and Loss account.
Gross Sales $802,071.00
Less — Returns and Allowances . 17,200.00
$151,595.00
Problem 75
A AND COMPANY
TRIAL BALANCE
December 31, 1920
Cash $ 90.00
Notes Receivable 1,250.00
Accounts Receivable 27,500.00
Furniture and Fixtures 2,250.00
NotesPayable $ 3.600.00
Accounts Payable 10.000.00
^50 ILLUSTRATIVE PROBLEMS
Inventory, January 1, 1920:
Dept. A 12,500.00
Dept. B 8,000.00
Dept. C 9.000.00
Purchases
Dept. A 37,000.00
Dept. B 10,500.00
Dept. C 13,500.00
Bank Overdrafts 2,800.00
Sales:
Dept. A 43,500.00
Dept. B 13,750.00
Dept. C 16,500.00
Wages
Dept. A 1,750.00
Dept. B 750.00
Dept. C 950.00
Trade Expense 1,500.00
Office Salaries 2,250.00
Rent, Taxes, Insurance 1,300.00
Bad Debts 430.00
Discounts Allowed Customers 630.00
A, Capital 25,000.00
B, Capital 20,000.00
A, Drawings 2,500.00
B, Drawings 1,500.00
$135,150.00 $135,150.00
Dept. A $15,000.00
Dept. B 9,000.00
Dept. C 8,000.00
Solution to Problem 75
A AND COMPANY
TRADING STATEMENT
Year Ended December 31, 1920
B— Interest $ 800.00
B—Balance of Profits. . 1,117.78 1,917.78
$11,800.00 $11,800.00
Problem 76
Real Estate:
Store No. 1 $4,000.00
Store No. 2 5,000.00
Reproductive Sound
Values Values
Buildings:
Store No. 1 $ 3,000.00 $2,000.00
Store No. 2 7,500.00 5,000.00
Inventories (Merchandise):
In Storeroom 14,000.00
In Storage (Butter and Eggs) 5.000.00
In Stores 11,000.00
No. 1 $ 2,300.00
No. 2 1,950.00
No. 3 3,135.00
No. 4 2,365.00
No. 6 1,250.00
On buildings, 5%.
On furniture and fixtures, 10%.
On automobiles, 25%.
Part of store No. 2 building is used for the general
oflBce and storeroom.
I?
258 ILLUSTRATIVE PROBLEMS
Solution to Problem 76
(a)
B 25,000.00
C 20,000.00
Vendor 46,000.00
Sundry Liabilities 46,000.00
For sundry liabilities taken over from John B.
Green and assumed by the company:
Trade Creditors $22,280.00
Wages 795.00
Mortgage Payable 7,500.00
Notes Payable 15,000.00
Interest on Mortgage 225.00
Interest on Note 100.00
Accrued Taxes (estimated) 100.00
Cash 31,500.00
Capital Stock Subscriptions 31.500.00
John B. Green Account $21,000.00
50% additional 10,500.00
Distributed as follows:
A, 42% of $30,000 $12,600.00
B, 42% of 25,000 10,500.00
C, 42% of 20,000 8,400.00
$31,500.00
Note:
Total subscriptions A, B, and C . $75,000.00
Required total payment 31,500.00
(b)
CASH ACCOUNT
Balance, July 1 $ 1,500.00 Creditors $39,660.00
Stock Subscriptions 31,500.00 Pay-Roll 1,572.50
Cash Sales 25,000.00 Interest on Mortgage . . 225.00
Received from Dealers . 4,200.00 Postage 10.00
From Stores 150.00 Balance 20,947.50
Over-Cash 15.00
Rental 50.00
$62,415.00 $62,415.00
o o o ® S © © ©
o q o o '^ q © q
*? «o 9 o «o d 6 'o
o
d >c •0
2 hj OS
* OS t- ,^
* OS CO >o 00
©^
a *< o OS 1—1
O «© 05 €6- «&
o
o oo o ©
?>
q q © q
V a 6 d q»c «o d
OS oo -* 00 t-
d
j>
d
t~
^ a »» —1 CO 00 »j * *
^^ ۩
1-H
«©•
^
«©
•w
/-"^s
U3
o tn
65
^ •O
OS
0,
^
©
^
«s o o § CO c CO
2 ° -H Ol ai 22 —I
o o
-i-i
CC ,^^
O ,^ o
o © o © ©
© c
c
©
q q q
% )-3
d
oo
«5 «s >d
so >o
<xi
wj
IQ
d »r
© «
«5
«5
us « »0 ^ f
O CO
O
li
'"
o
tr~
FM
IS*
»>
©^
so" eo'
o «©• OS «& 6&
O
<t3 o o o © © © c ©
©
o q q © © q c
"S-S-o d «c «o «o ui d « «5
(X «o ^ «5
1-1 © -^
«
*
*** "1 «o Tf. t- © «o
ifs" »r d" •* of 0« -"S to
o « ot
۩
o«
«©
tn
ni _ CO o o o
o ©
© ©
© ©
© c ©
©
Sail
o q c
Cash
d d
o d
o d
© d
© d
© c d
©
Register o
O «5 o »c
c
c ©
®.
Gross per CO « cc ui of »o «f ©*
o< eo
ǩ ۩ ۩
t»
V
1-
CO
*J
o "rt
CO
:n
fc
H
-H o» ISO •* »o
6 6 6 6 6 -1
;?: z iz; ;z: z ^c 1
262 ILLUSTRATIVE PROBLEMS
Profit and Loss Account — Continued
Dr. Cr.
$2,340.00 $2,340.00
$ 427.09 $ 427.09
• This may be written off over a period of, say, six months, $100 monthly.
ILLUSTRATIVE PROBLEMS 263
Journal entry:
(d)
Cash 50.00
John B. Green 50.00
To record receipt of rent in arrears.
Merchandise 35,000.00
Accounts Payable 35,000.00
To record purchase of merchandise.
264 ILLUSTRATIVE PROBLEMS
Accounts Payable 750.00
Merchandise 750.00
To record return of merchandise.
Buildings:
$ 2,000.00 Store No. 1 $ 2,000.00
7,000.00 5,000.00 Store No. 2 5,000.00 7,000.00
Liabilities
July 1, 1914 July 31, 1914
$100,000.00 Capital Stock— Authorized Issue . . $100,000.00 $100,000.00
Unpaid Subscribed (per contra) . . . 48,300.00
Accruals:
$ 100.00 Interest on Notes Pay-
able $ 150.00
225.00 Interest on Mortgage . 37.50
4«5.00 100.00 Taxes (estimated) ... 116.66 304.16
(e)
Netincrease $800.00
Problem 77
The Brown Manufacturing Corporation, finding its
credit impaired, prepares the following statement as of
January 1, 1908:
Assets Liabilities
Plant $248,607.00 Capital Stock $200,000.00
Merchandise 345,156.00 Accounts Payable .... 488,361.00
Accounts Receivable . 174,216.00 Profit and Loss 80,685.00
Cash 1,067.00
$769,046.00 $769,046.00
ILLUSTRATIVE PROBLEMS 267
Solution to Problem 77
(a)
January 1, 1908
Receivers $630,657.00
Plant $248,607.00
Merchandise .^ 241,610.00
Accounts Receivable 139,373.00
Cash 1,067.00
The above entry is made to clear the books of
the above assets and to charge the receivers
with them at the value at which taken over.
268 ILLUSTRATIVE PROBLEMS
Accounts Payable 488,361.00
Receivers 488,361.00
To credit the receivers with the liabilities placed
in their hands.
Balance Sheet
Assets Liabilities
Receivers $142,296.00 Capital Stock $200,000.00
Deficit 57,704.00
$200,000.00 $200,000.00
(b)
October 1, 1909
Plant $241,747.00
Merchandise 77,564.00
Accounts Receivable 63,132.00
Cash 47,637.00
Receivers $430,080.00
To bring on the books the assets turned over
to the company by the receivers at the close
of the receivership.
Receivers 186,883.00
Accounts Payable 186,883.00
To bring on the books the amount due creditors
as reported by the receivers.
Receivers 100,901.00
Surplus 100.901.00
To close the balance of the Receivers account
into Surplus,as this balance represents the net
addition to net worth due to the receivership.
ILLUSTRATIVE PROBLEMS 269
(c)
BALANCE SHEET
October 1, 1909
Assets Liabilities
Cash $ 47,637.00 Capital Stock $200,000.00
Accounts Receivable . 63,132.00 Accounts Payable .... 186,883.00
Merchandise 77,564.00 Surplus 43,197.00
Plant 241,747.00
$430,080.00 $430,080.00
(d)
January 1, 1908
Plant $248,607.00
Merchandise 241,610.00
Accounts Receivable 139,373.00
Cash 1,067.00
Brown Manufacturing Corporation $630,657.00
To bring on the receivers' books the assets
taken over from the Brown Manufacturing
Corporation.
(e)
October 1, 1909
Problem 78
Assets Liabilities
Cash $ 12,188.00 Accounts Payable $ 62,060.00
Accounts Receivable . . 71,227.00 Notes Payable 60,000.00
Investments 13,950.00 Capital Stock 50,000.00
Inventory 83,312.00 Surplus 55,497.00
Fixtures 46,880.00
$227,557.00 $227,557.00
Accounts receivable:
Contracted in 1912 $51,822.00 estimated to shrink $500
" 1911 5,715.00 " " " 25%
" 1910 3,180.00 75%
prior to 1910 10,510.00 all bad
$46,880.00
Solution to Problem 78
Surplus 793.00
Cash 793.00
To remove from the Cash account items of ex-
pense vouchers which have been carried as cash.
Surplus 14,823.75
Reserve for Bad Debts 4,313.75
Accounts Receivable 10,510.00
To write off estimated shrinkage in accounts re-
ceivable.
Surplus 13,950.00
Investments 13,950.00
To write off the investments which are considered
of no value.
i8
274 ILLUSTRATIVE PROBLEMS
Surplus 7,525.00
Inventory 7,525.00
To remove from the Inventory account the un-
salable stock of no value.
Surplus 20,082.70
Reserve for Depreciation 20,082.70
The above entry is to create a reserve for depre-
ciation of fixtures based upon a life of ten years.
Purchases of fixtures and corresponding deprecia-
tion are as follows:
$46,880.00 $20,082.70
Surplus 3,512.00
Accounts Payable 3,512.00
To bring on the books the corresponding liability
for goods bought and included in the inventory.
Assets Liabilities
Cash $ 11,145.00 Accounts Payable $ 65,572.00
Accounts Receivable . 71,227.00 Notes Payable 60,000.00
Advances to Salesmen. 250.00 Reserve for Bad Debts 14,823.75
Inventory 75,787.00 Reserve for Deprecia-
Fixtures 46,880.00 tion 20,082.70
Deficit 5,189.45 Capital Stock 50,000.00
$210,478.45 $210,478.45
ILLUSTRATIVE PROBLEMS 275
$219,779.00 $219,779.00
Comments on Problem 78
Deduct:
Operating Expenses $175,000.00
Bad Debts 1,750.00
Depreciation of Fixtures 4,688.00
Extinguishment of Reorganization Expenses . . 5,000.00 186,438.00
Dividend Charges:
Preferred Stock, 6% of $75,000 $ 4,500.00
This leaves net profits belonging to common *
Problem 79
Several manufacturers consolidate their interests
and organize the Consolidated Manufacturing Com-
pany with an authorized capital stock of $1,000,000,
divided into 5,000 shares of common stock and 5,000
of preferred stock at $100 each par value.
The manufacturers sell to the company all of their
Solution to Problem 79
Cash $ 1,000.00
Common Stock $ 1,000.00
10 shares common stock issued for cash.
Vendors 1,000,000.00
Cash 1,000.00
Common Stock 499,000.00
Preferred Stock 500,000.00
Payment of balance due on purchase con-
tract.
Cash 180,000.00
Discount on Bonds 20,000.00
Stock Bonus 100,000.00
Treasury Common Stock 100,000.00
Bonds 200,000.00
Sale of bonds with treasury stock bonus.
(a) Opening Entry for Corporation. The opening entry for the
corporation is described in Volume I, Chapter XXXIII,
§8.
(b) Plant and Sundry Assets Account. The use of such an ac-
count is described in Volume I, Chapter XXXIII, § 8.
It is also commented upon in Problem 67.
(c) Valuation of Good-Will. The various methods of valuing
good-will are described in Volume IV, Chapter XIX, § 4.
280 ILLUSTRATIVE PROBLEMS
o
o o
o
o o
o
o >c
o
^5
o g
° cs
<u
a
o
y^ a
I— ^ w cS to
fl
I
a
2 I
o
p 1-3 V a s u a ** o
H 9 *« a ^
m
U
P
J
w o
o
o
o o
o
o o^ o^
X5 «5 O
00 t~ o
Q
O
o
u
oi
<3 03
<! T3
W
^ -a
ILLUSTRATIVE PROBLEMS 281
Problem 80
Solution to Problem 80
Cash 50,000.00
Subscribers to Preferred Capital Stock. 50,000.00
To record payment of subscriptions.
Comment on Problem 80
No entry can be made for materials and supplies
since no price has yet been determined upon.
The account with Plant and Sundry Assets will
remain upon the books until it is determined what
values shall be assigned to each property item taken
over. At that time an entry will be made debiting
each of the asset units taken over and crediting the
Plant and Sundry Asset account. The posting of this
entry will open the asset accounts upon the books.
Problem 81
$355,000.00 $150,000.00
Together $505,000.00
Together $ 32,500.00
Solution to Problem 81
(a)
(b)
Surplus 20,000.00
Common Stock 20,000.00
To show the distribution of a stock dividend to
stockholders of Smith Company to an
amount equal to the difference between
. $100,000 new issue and that issued to Jones
Company stockholders.
Cash 100,000.00
Five Per Cent Bonds 100,000.00
To show sale of $100,000 bonds for cash.
Problem 82
The X
Corporation is formed with a capital stock
of $500,000 (of which $200,000 is preferred and $300,000
is common stock) and consolidate three
to acquire
existing corporations designated as A, B, and C and
having the following status respectively:
Book
Liabilities Surplus Deficit Capital
Accounts
19
290 ILLUSTRATIVE PROBLEMS
Solution to Problem 82
(a)
The X
Corporation incorporated under the laws of the
State of , with an authorized capital stock of
Cash $100,000.00
Plant and Sundry Assets 444,000.00
Organization Expense 100,000.00
Good- Will 39,000.00
Sundry Liabilities $183,000.00
Capital Stock, Preferred 200,000.00
Capital Stock, Common 300,000.00
The opening entry of this company, explained
as follows
—
Cash Working Capital supplied by
Promoter $100,000
Liabilities
A Company $ 56,000
B Company 80,000
C Company 47,000 $183,000
ILLUSTRATIVE PROBLEMS £91
THE X CORPORATION
OPENING BALANCE SHEET
As at
$683,000.00 $683,000.00
(b)
A COMPANY
CLOSING ENTRIES
Good- Will $ 10,000.00
Surplus $ 10,000.00
Excess over book value to be paid.
Surplus 25,000.00
Capital Stock 100,000.00
Vendee Company Preferred Stock 62,500.00
Cash 62.500.00
To close all accounts and distribute Vendee
Company shares and surplus cash among
stockholders.
292 ILLUSTRATIVE PROBLEMS
(c)
PROMOTER'S TRANSACTIONS
Plant and Sundry Assets (including Good- Will) . . . $483,000.00
Sundry Assumed Liabilities $183,000.00
A Company, Vendor 125,000.00
B Company, Vendor 100,000.00
C Company, Vendor 75,000.00
To record net assets bargained for and trans-
ferred to the promoter.
Cash 240,000.00
Promotion Profit and Loss 90,000.00
— Vendee Company
Capital Stock, Preferred 30,000.00
Capital Stock, Common — Vendee Company 300,000.00
To record sale to bankers of full common stock
issue at 80%, difference being cost of under-
writing; with additional bonus of one share
of preferred stock to each ten shares of com-
mon stock.
Cash 10,000.00
Capital Stock, Preferred — Vendee Company 10,000.00
To record shares taken by partners at par, per
agreement.
$100,000.00 $100,000.00
Balance $ 10,000.00
Comment on Problem 82
The balance of the Promotion Profit and Loss
($10,000) represented by 100 shares preferred capital
is
Problem 83
Current
Cash $ 25,000.00
Accounts Receivable 350,000.00
Notes Receivable 1,000.00 $ 376,000.00
$ 100,000.00
Merchandise, Materials, and
Supplies 250,000.00 350,000.00
Investments:
Acme Company Bond $ 1,000.00
Stock in Other Companies. . 200,000.00 201,000.00 $ 927,000.00
Plant:
Real Estate, Main Plant . . . $1,000,000.00
—
Less Reserve for Depre-
ciation 50,000.00 $ 950,000.00
$3,152,000.00
Good- Will:
Good- Will Account $1,000,000.00
Patent Rights 600.000.00
Organization Expense 100,000.00 $1,700,000.00
Deferred Charges:
Unexpired Insurance $ 20,000.00
Prepaid Interest 5,000.00
Personal Advances 2,000.00 27,000.00
Deficit 3,016,000.00
$8,195,000.00
ILLUSTRATIVE PROBLEMS 295
Liabilities
Current:
Accounts Payable $ 300,000.00
Accrued Wages 10,000.00
" Taxes 5,000.00
Commissions 10,000.00 $ 325,000.00
$1,125,000.00
Capital Stock:
Preferred $2,000,000.00
Common 5,000,000.00 7,000,000.00
Reserves
For Completion of Contracts 70,000.00
$8,195,000.00
$4,250,000.00
X Y Z COMPANY
BALANCE SHEET
Assets
Current
Cash $ 107,800.00
Accounts Receivable 350,000.00
Notes Receivable 1,000.00 $ 458,800.00
$ 600,000.00
'
Solution by Hazen P. Philbrick, C. P. A.
ILLUSTRATIVE PROBLEMS 297
Investments:
Acme Company Bonds .... $ 1,000.00
Stock in Other Companies . 200,000.00 201,000.00 $1,109,800.00
Plant:
Real Estate, Main Plant . . . $1,000,000.00
Less —
Reserve for Depre-
ciation 50,000.00 $ 950,000.00
$3,334,800.00
Good- Will:
Good- Will Account '
$1,000,000.00
Patent Rights 600,000.00
Organization Expense 137,200.00 $1,737,200.00
Deferred Charges:
Unexpired Insurance $ 20,000.00
Prepaid Interest 5,000.00
Personal Advances 2,000.00 27,000.00
$5,399,000.00
Capital Stock:
Preferred Authorized $2,000,000.00
Less— Unissued 40,000.00 $1,960,000.00
$5,399,000.00
REORGANIZATION ADJUSTMENT
Deficit Account (old company) $3,016,000.00
Reorganization Expense (old company) 100,000.00
Reduction in Common Stock $3,750,000.00
Preferred Stock Surrendered, not reissued,
(20% of 10% of $2,000,000) 40,000.00
Estimated Profit Accrued on Work in
Progress 100,000.00
Balance considered a Reserve for Working
Capital 774,000.00
$3,890,000.00 $3,890,000.00
ILLUSTRATIVE PROBLEMS 299
REORGANIZATION CASH
Received from Preferred Stockholders ($15 per
share on 18,000 shares) $ 270,000.00
Received from Common Stockholders ($5 per
share on 50,000 shares) 250,000.00
Paid on Notes Payable $ 300,000.00
Reorganization Expense (new company) 137,200.00
$7 per share on 18,000 shares $126,000.00
. .
$ 520,000.00 $ 520,000.00
MORTGAGE BONDS
Sold for cash to Preferred Stockholders $ 270,000.00
Sold for cash to Common Stockholders 250,000.00
Applied at par on Notes Payable 240,000.00
Applied at par on Accounts Payable 240,000.00
Comments on Problem 83
Good-Will account is not closed out. Though its
value may be problematical, it may as well be taken
over with the rest of the assets at book value. On the
other hand, it would not be proper for the new company
to carry as an asset the expense of organization of a
defunct company hence, the account is charged off.
;
—
the reorganization an organization expense.
In actual cases the balance of such a reorganization
300 ILLUSTRATIVE PROBLEMS
Problem 84
A is an operating company and B is a holding com-
pany. The following statements are taken from the
books of the respective companies, viz.:
A COMPANY
Assets
Cash on Hand $35,000.00
Book Accounts Receivable . 25,000.00
ILLUSTRATIVE PROBLEMS 301
$1,578,700.00
Liabilities
$1,578,700.00
B COMPANY
Assets
Investments:
A Company's Stock $500,000.00
Other Investments 500,000.00 1,000,000.00
$2,292,000.00
Liabilities
$2,292,000.00
Solution to Problem 84
WORKING SHEET
Eliminations Consolidated
A B and Balance
Company Company Additions Sheet
Assets
Cash $ 35,000.00 $ 14,000.00 $ 49,000 00
25,000.00 6,000.00 31,000 00
21,000.00 21.000 00
7,000.00 7.000 00
15,000.00 15,000.00
Premium on Sinking Fund Bonds. 700.00 700.00
45,000.00 $ 45,000.00
25.000.00 25,000.00
500,000.00 500,000.00
5,000.00 600,000.00 505,000.00
1,400,000.00 1,250,000.00 2,650.000.00
$3,278,700.00
Liabilities
$ 12.000.00 $ 7,000.00 $ 19,000.00
Bills Payable 50,000.00 130.000.00 180,000.00
8,000.00 8,000.00
12,000.00 10,000.00 22,000.00
65,000.00 65.000.00
Bonds 750.000.00 1.100,000.00 1.850,000.00
45,000.«0 $ 45,000.00
Capital Stock 500.000.00 1.000,000.00 525.000.00 975,000.00
181.700.00
Deficit 22,000.00 159.700.00
$3,278,700.00
ILLUSTRATIVE PROBLEMS 303
A AND B COMPANIES
CONSOLIDATED BALANCE SHEET
Assets
Cash $ 49.000.00
Accounts Receivable 31,000.00
Stock Inventory 21,000.00
Investments 505,000.00
Plant, Franchises, etc 2,650,000.00
Fund Trustee
Sinking 15,000.00
Premium on Sinking Fund Bonds 700.00
Prepaid Accounts 7,000.00
$3,278,700.00
$3,278,700.00
304 ILLUSTRATIVE PROBLEMS
Problem 85
Fromthe following three trial balances prepare a
consolidated balance sheet as at December 31, 1912, in
the form you would draw it up for presentation to the
stockholders of the parent company (the Safety Razor
Company), showing as separate items therein, (a) the
total good-will of the combined companies; and (b) the
net profits accruing to the new corporation, viz.: to
the Safety Razor Company.
$3,120,000.00 $3,120,000.00
L W COMPANY
TRIAL BALANCE
At December 31, 1912
$1,510,000.00 $1,510,000.00
$875,000.00 $875,000.00
30
306 ILLUSTRATIVE PROBLEMS
$ 810,000.00
Less — Dividend declared 100,000.00
$ 65,000.00
Profits for nine months from March 31 to December 31, 1912 100,000.00
Premium 1,700.000.00
Solution by A. E. Andersen, C. P. A.
308 ILLUSTRATIVE PROBLEMS
SAFETY RAZ
CONSOLIDATED
L W Company
(After Posting Adjustment
No. 1)
Assets Liabilities
Intercompany Balances:
L W Company
$ 175,000.00
Steel Blade Company.
(1) 20.000.00
Safety Razor Company 100,000.00
Intercompany Stockholdings, par value:
L W Company 400,000.00
Steel Blade Company $200,000.00
Intercompany Holdings, premium over par:
L W
Company and Steel Blade Company
Steel Blade Company Cost — 100,000.00
Steel Blade Company Written up — 100,000.00
Good-Will 250,000.00
Surplus:
Balance at January 1, 1912 605,000.00
ProfitJanuary 1 to March 31, 1912 30,000.00
Surplus — Writing up Steel Blade Stock 100,000.00
Profits April 1 to December 31, 1912 (1) 10.000.00 75,000.00
Dividends declared April 1 to December 31, 1912. 100,000.00
Dividends received April 1 to December 31, 1912.
(1) 10,000.00
Inventories
250,000.00
Cash 90,000.00
Accounts Receivable. . . 195,000.00
Property Equipment. . . 325,000.00
Accounts Payable 125,000.00
Organization Expense. .
Administration Expense.
Capital Stock, Preferred.
Capital Stock, Common.
$1,630,000.00 $1,630,000.00
Note: To save space the adjustments Nos. 1 and 2 have been included in the same columns as the
ILLUSTRATIVE PROBLEMS 309
OR COMPANY
WORKING PAPERS
S.\FETY Razor Company
Consolidated Balance
Steel Bl.u)e Comp.vxy (After Posting Adjustment
Sheet
1
No. 2)
1 $195,000.00 $ 100,000.00
/ 150,000.00
it
1 : ....}
<t $150,000.00
4 400,000.00 \
K 600.000.00 400,000.00 /
A 1,700.000.00
7
8
9 50,000.00
$1,530,000.00
10 50,000.00
11 15,000.00
^9
IS 100.000.00 (2) 40,000.00 $ 125,000.00
"1
14
15 $ 100.000.00/
r
16
\ 190,000.00 450,000.00
17 10,000.00 270.000.00 370,000 00
18 105,000.00 300,000.00
''9 325,000.00 650,000.00
W 90.000.00 20,000.00 235,000.00
f1 75,000.00 (2) 15,000.00 60,000.00
99 25,000.00 (2) 25,000.00
r^ 1,500,000.00 1,500,000.00
94 1,500,000.00 1,500,000.00
trial balance figures, but the items have been kept separate and marked with the numberof theadjustment.
310 ILLUSTRATIVE PROBLEMS
ADJUSTMENTS
Profit and Loss, Mar. 31 to Dec. 31, 1912 (Rent) $ 10,000.00
Inventory (Merchandise in transit) 10,000.00
Steel Blade Company $ 20,000.00
To adjust Steel Blade Co. current account.
and Loss, March 31 to December 31, 1912
Profit 40,000.00
Organization Expense 15,000.00
Administration Expense 25,000.00
To write off administration and a portion of
organization expenses, balance of latter car-
ried forward as deferred charge.
Liabilities
Capital Stock Issued and Outstanding:
Preferred Stock $1,500,000.00
Common Stock 1,500,000.00 $3,000,000.00
Current Liabilities:
Accounts Payable 235,000.00
Surplus:
Net Income from March 28 to December 31, 1912 125,000.00
$3,360,000.00
ILLUSTRATIVE PROBLEMS 311
Comments on Problem 85
Problem 86
On January 1, 1913, the A B Company owned 90%
of the stock of the X Y Company and 80% of that of
the P Q Company, two subsidiary companies which it
TRIAL BALANCES
At December 31, 1913
$ 85.000.00 $ 75,000.00
Good-Wll $100,000.00
Stockholdings:
In X y Company :
•After crediting the proceeds of the 100 shares sold, prior to which the investment had been valued
at cost.
fDividends received from subsidiary companies, less expenses of parent company.
:§? •
o
© o © ©©©
oo© ©
©©© »c
•3 a a
© © ©
o ©o © ©
o
5?
© 'O © >o ©
©©
©^©^13* © t- »o
© 00©»
00 *
4) .5
aw
c/2
tf w
*
-Jl
hH
W
r^)
s C5
g I—
1—
a> OS
rH
Q O
I-H
oc
2 00
a< t^ :s
4^
-51
Q S
Q
c
>^ w
;^ H
4-i < <J Q
3 p^ 1^
"p ^
C/J o o
u
m O a © ©
S
o ©©
© ©
O"
_:
•
«:«
to >» ca -^ «3 5 ea
C
j>. 4)
Ph . a g- a
•
eS a>
3 " fc a a a
-?;. -pT3
a
o 2 o
O § O 00 I :
03 tJOQ
ILLUSTRATIVE PROBLEMS 315
o
o o o
© ©o
oo© ©©©©
©
s a a a
5 c
2 o ©
© © oc>©©©©©©
©©© «liO©©©«5»o©
^'3*©«5©l>l>©
^H ^"3 0) C-^JS .^ o
H ^
a, ©
©©©
S
o © o
- Q
a c
a a .2
a
o >! ^ 60 ^
X ^1
.£ <a c
V.O Ji 'Z.
c 5 S
i :;^,
n '=.£>„._ „
g J) cs c "O J'
©
© o o g g » - s-s
©_^ c o c
GO
>1 is '€ ii
O 3 o
S §• a- -^
a a 2
. cS ro rj "3 S a
J" a,
n . « M J? D. '^
5S
a •• a a c .i:
.2 -o .2
e
8
o ° a
»5o<«o'»'^4^Sca±; a
o o e J
< c
a ;i ca
^ c O vj o *
uU o a «
fciU ui2 '^uUr^ o O "a
°
2-
" 'a
Ei
-S ii --
-r
§
PQ S .2 S £ -^So
l^' -S ^ ^ "u -a c: "tc 3
O
316 ILLUSTRATIVE PROBLEMS
A B COMPANY AND SUBSIDIARIES
CONSOLIDATED BALANCE SHEET
December 31, 1913
Assets
Capital Assets:
Properties $160,000.00
Good- Will 73,250.00
Current Assets 357,000.00
$590,250.00
Liabilities
Current Liabilities:
Accounts Payable 165,000.00
Surplus:
Balance at January 1, 1913 $ 50,000.00
Add — Net Profits for year ending December 31,
1913 60,250.00
$110,250.00
Deduct— Dividends Paid 30,000.00 80,250.00
$590,250.00
r Comments on Problem 86
Problem 87
A
"blank" firm is engaged in the lumber business
owning timber lands in fee and licensed, saw mills, and
other equipment, and 90% of the stock in another lum-
ber corporation. They instruct an accountant to ex-
amine their accounts for the purpose of ascertaining the
true financial position. The following is a trial balance
from the firm's books December 31, 1914, after closing:
Dr. Cr.
Cash $ 20,000.00
Accounts Receivable 150,000.00
Logs and Manufactured Lumber 200,000.00
820 ILLUSTRATIVE PROBLEMS
Advances on Account of Season's Logging Dr. Cr.
Operations 100,000.00
Investments in Controlled Company, 900
shares (cost) 99,000.00
Timber Lands (cost) 500,000.00
Mill Plants and Equipment (cost) 250,000.00
Loans Payable $ 150,000 dC
Accounts Payable 250,000.00
Deposits on Orders 50,000.00
Mortgage on Plants 150,000.00
Controlled Company Current Account 20,000.00
Partners' Capital 739,000.00
$1,339,000.00 $1,339,000.00
Cash $ 5,000.00
Accounts Receivable 70,000.00
Logs and Manufactured Lumber 40,000.00
Timber Lands 50,000.00
Mills 100,000.00
BillsPayable $100,000.00
Blank Firm 30,000.00
Accounts Payable 20,000.00
Capital 100,000.00
Surplus 15,000.00
$265,000.00 $265,000.00
21
322 ILLUSTRATIVE PROBLEMS
Cash $ 20,000.00
Accounts Receivable 150,000.00
Logs and Manufactured Lumber 200,000.00
Advances on Logging Operations 90,000.00
Investment in Controlled Company (900
shares), at cost 99,000.00
Timber Lands, appraised value 600,000.00
Mill Plants and Equipment, at cost 250,000.00
Controlled Company, Current Account 30,000.00
Loans Payable $ 150,000.00
Accounts Payable 250,000.00
Deposits on Orders 50,000.00
Mortgage on Plants 150,000.00
Partners' Capital 837,400.00
Reserve for License Fees 1,600.00
$1,439,000.00 $1,439,000.00
$277,000.00 $277,000.00
$1,587,000.00
(d) Report:
October 15, 1915
Blank Firm,
Dear Sirs :
Comments on Problem 87
There are several matters in the course of this solu-
tion upon which differences of opinion may arise
The adjustment to bring book value of timber lands
up to the appraised value might have been credited to
a special account of some sort, in each set of books, but
in the author's opinion nothing is thereby gained. If
the lands of the firm are really worth $600,000, the real
present net worth of the firm (which is all that partners'
capital stands for) is $839,000, and no useful purpose
is served by having this figure divided between two
326 ILLUSTRATIVE PROBLEMS
Cash $311.05
Accounts Receivable 543.27
Accounts Payable 10.52
Due to Treasurer 843.80
Due Account
to Treasurer $200.00
Accounts Receivable $200.00
Cash collected by treasurer from on June 30.
WORKING SHEET
Peerless Storage
Warehouse Co. Elimina-
Y tions of
Total
Company Inter-
Treasurer's Warehouse Relations
Books Books
Assets
Cash $ 98.05 $311.05 $ 24,000.00 $ 24.409.10
Accounts Receivable 343.27 343 27
Investments 48,803.64 19,650.00 68,453.64
Warehouse Property 50,000.00 50,000.00
Land 65,000.00 65 000 00
Prepaid Interest 81.99 81 99
Accrued Interest Receivable 1,584.00 1,584.00
Due from Warehouse 643.80 $ 643.80
Peerless Storage Warehouse Com-
pany Stock, at par 50,000.00 50,000.00
Premium paid for Peerless Storage
Warehouse Company Stock 500.00 500.00
Liabilities
Accounts Payable $ 10.52 $ 10 52
Accrued Bond Interest $ 850.00 $ 312.50 1 162 50
Accrued Taxes 200.00 200 00
Mortgage Bonds 42,500.00 50,000.00 92,500.00
Capital Stock 55,000.00 100,000.00 $50,000.00 105,000.00
Reserve for Depreciation 497.24 497.24
Surplus 2,364.24 8,637.50 11,001.74
Due to Treasurer 643.80 643.80
Problem 89
A cloak manufacturing concern, turning out but one
grade of cloaks, claims to have been robbed on the
night of April 16 and forthwith files a claim under a
burglary insurance policy it was carrying.
The proof of the loss filed by the assured contained
two items, viz.: 300 cloaks valued at $12,000 and 1,000
yards of silk stated to be worth $1,500.
The insurance company being notified of the loss,
immediately ordered an inventory to be taken, which
was done on the morning of the 17th, and disclosed the
following
1,250 cloaks
6,250 yards of cloth —
_^
o
o o o
o o
o o
o ©
o o
o O O
o
o o
CS
*J d
«5
d d
o d
»o
d
"O
«5
l>
»o'
J>
>d
l>
d
o
«o
o o "O^ **! °°- <s» o *l
rH O^ «5
H wT d
1—
05
rH
<« eo d eo"
eo
»f
i> eo
۩
1
1
^ >o
1
«&
eo
1
«© 1 »i
c
c
>
>
o
o o
o o
o o
o o
o
4-1
V
9.(
) »o
(V
d
o «5
<s«
«5
t»
d
«5
o 4 1—1 O 1-1 eo t^
X (y (« o
i-i
eo
i-i
C> 1 eo"
۩
s €^> «© 1*
^ c
«r
) o
»o
c
c
>
>
o
«5
o
«5
o
o
. C-t
O. (M c *1 l~ »o
^>H
53
«C d" c
J,
d >.
J>
2|
o c o
o c > o
o c o
o
o c c c
05 d c d c d c d
o
o o c
« o c
c
o c
u o^ •o •o c
«
•o
M ^H 9J (« «r 00 eo"
€© «e- €& «©•
00
m
m
00
O c o c o c o
o c o c o c o
O O
u
c3 o^ ur »o c »o_ c
«
»o
Mo h-5 ^>^ Oi eo If 00* eo"
u P^
O o c o
o c o
o c o
o c c c o
fe
-*J
CA
d
«5
»r
«
«5
t-
c
w
«5
o<
c
c
»c
»»
O m
O «o «C T-l O) * « 05
o M
<;
»r C eo
«&
a
e<-
i-T
«5
«©•
»r
»)
•o
o«
«&
P5 O 66-
Ph ll
o O
O C
IT
o
•c
c
c
o
«c
c
c
o
«o
s SO Oi «s «• o c o
3 1— f o r eo
7^ i>
"^
i
5
o
<->
2
"3 eo
< as a o OS
-a
s n
p.
o
M 03 IS
60
'3 60 «
0)
O] 2 3 ]3
_C 3
03
& C x
a '3 o >-s »
Ji O h
o 3 >> -o V
o ft c a
S o
s 2 03 3 3 >>
-a -a o
^ 03
o
o
> O 3
a 3 03
o
a
4-1 -a
o O ft
c
a 1
-0
1
o
2 2 OT u en
a t 3 3 > 5
O 5 T3 -a
o PH > -3
O frH
<! Q « Oh 1
334 ILLUSTRATIVE PROBLEMS
CLOAK MANUFACTURING COMPANY
STATEMENT OF BURGLARY LOSS
Night of April 16, 1913
Purchases since:
Cloth, 18,750 yards at $.50 $ 9,375.00
Silk, 5,000 yards at $1 5,000.00
$57,550.00
Deduct — Cost of Goods Sold:
4,500 cloaks at $8.50 38,250.00
Discrepancy $ 500.00
Cloaks:
As per proof of loss (300 at $40) $12,000.00
At cost (300 at $8.50) 2,550.00
Silk:
Cash $ 7,500.00
Accounts Receivable 45,000.00
Notes Receivable 1,875.00
Inventory:
Raw Materials $20,000.00
Labor 30,000.00
Manufactured Goods 16,250.00 66,250.00
$120,625.00 $120,625.00
Wages 10,000.00
Traveling Expenses, Commission, etc 26,250.00
Salaries 19,000.00
Rent 3,750.00
Bad debts 6,375.00
Depreciation 1,500.00
Interest 625.00
Sales 250,000.00
Return Sales 7,500.00
Material $45,000.00
Labor 80,000.00
Solution to Problem 90
Cash $ 7,500.00
Accounts Receivable 45,000.00
Notes Receivable 1,875.00
Inventory, beginning:
Raw Material 22,500.00
Labor 32,500.00
Manufactured Goods 55,000.00
Purchases during period 50,000.00
Labor 87,500.00
Wages 10,000.00
Traveling Expense, Commission, etc 26,250.00
32
338 ILLUSTRATIVE PROBLEMS
Salaries 19,000.00
Rent 3,750.00
Bad Debts 6,375.00
Depreciation 1,500.00
Interest 625.00
Sales $250,000.00
Return Sales 7,500.00
Accounts Payable 5,875.00
Notes Payable 20,000.00
Capital Stock 80,000.00
Surplus, January 1, 1915 21,000.00
$376,875.00 $376,875.00
Raw Manufactured
Total Material Labor Goods
Inventory at beginning . . $110,000.00 $22,500.00 $ 32,500.00 $55,000.00
—
Add Purchases and
Labor 137,500.00 50,000.00 87,500.00
Deduct — C onsumption
per Cost Records 163,750.00 45,000.00 80,000.00 (38,750.00)
Difference in Inventory,
December 31, 1915 ... $ 17,500.00 $ 7,500.00 $ 10,000.00
Net Profit for period, per Statement of Income and Profit and
Loss $11,250.00
ILLUSTRATIVE PROBLEMS 339
Sales $250,000.00
Less Return Sales 7,500.00
Cost of Sales:
Material
Inventory, beginning $ 22,500.00
Purchases during period 50,000.00
$ 72,500.00
Less — Inventory, end, per state-
ment 27,500.00 $ 45,000.00
Labor:
Inventory, beginning $ 32,500.00
Labor during period 87,500.00
$120,000.00
Less — Inventory, end, per state-
ment 40,000.00 80,000.00
Manufactured Goods:
Inventory, beginning $ 55,000.00
Inventory, end 16,250.00
Expenses:
Wages $ 10,000.00
Traveling Expense, Commissions, etc 26,250.00
Salaries 19.000.00
Rent 3,750.00 .
Interest 625.00
Bad Debts 6,375.00
Depreciation 1,500.00 67,500.00
Assets
Cash $ 7,500.00
Accounts Receivable 45,000.00
Notes Receivable 1,875.00
Inventory (per Statement)
Raw Material $27,500.00
Labor 40,000.00
Manufactured Goods 16,250.00 83,750.00
$138,125.00
$138,125.00
Problem 91
Sales $800,000.00
Operating Expenses:
Selling Expenses $ 50,000.00
General Administrative Expenses 70,000.00 120,000.00
Problem 92
Suspense 100.00
Sales 100.00
Proprietor 100.00
Wages 100.00
Suspense 1.50
Credit Account 1.50
Suspense Account
$155.56 $155.56
Chapter XXXIII.
ILLUSTRATIVE PROBLEMS 345
Problem 93
The a firm of traders, doing business in
office of
San Francisco, was destroyed by an earthquake. The
books of account, which had been fully posted and
which may be presumed to be correct, were badly dam-
aged. The following ledger accounts were found to be
legible: purchases, net, $69,000; discounts lost $640;
discounts gained $3,450; sales $54,000; bills receivable
$33,000.
Inquiry at the bank disclosed a balance on deposit
of $129,000. Bills receivable amounting to $45,000
had been discounted at the bank. An audit of the
checks paid by the bank showed that $99,000 had
been paid to creditors of the firm (including $60,000
notes payable).
A balance sheet prepared at the last closing of the
books was produced, containing the following items:
cash $60,000; accounts receivable $126,000; loans re-
ceivable $24,000; real estate $90,000; notes receivable
$78,000; capital $318,000; notes payable $60,000.
Prepare a trial balance supplying the missing
accounts and giving full particulars for each of
them.
Solution to Problem 93
Cash
Balance $ 60,000.00 Notes Payable . . . ... $ 60,000.00
Notes R e c e i V a ble Creditors 39,000.00
Discounted .... 45,000.00 Balance 129,000.00
Customers 123,000.00
$228,000.00
$228,000.00
Balance $129,000.00
346 ILLUSTRATIVE PROBLEMS
Customers
Balance $126,000.00 Discounts Lost $ 640.00
Sales . . 54,000.00 Cash 123,000.00
Balance 56,360.00
$180,000.00 $180,000.00
Balance $ 56,360.00
Creditors
$69,000.00 $69,000.00
Balance $26,550.00
Notes Receivable
Balance $78,000.00 Cash, Notes Receivable
Discounted $45,000.00
Balance 33,000.00
$78,000.00 $78,000.00
Balance $33,000.00
Notes Payable
Cash $60,000.00 Balance $60,000.00
Trial Balance
Real Estate $ 90,000.00
Cash 129,000.00
Notes Receivable . . 33,000.00
Loans Receivable . . . 24,000.00
Accounts Receivable 56,360.00
Accounts Payable . . $ 26,550.00
Capital 318,000.00
Sales 54,000.00
Purchases 69,000.00
ILLUSTRATIVE PROBLEMS S47
$402,000.00 $402,000.00
Problem 94
Schedule 1
Schedule 2
1,640 X 50
=
X .2223 $.46
40 X 1,000
45 X .00271 = $.12
$.1680.
The total cost of the piece is $89.25, and the total
cost per yard is $2.30 based on 38.7 yards to the
piece.
350 ILLUSTRATIVE PROBLEMS
^ fX
o >c
00
o
»o
«o
(X
o 05 eo «o 05
00
«• 99-
o> t~- «* on
t^ 50 MS
< o >o
eo
^ >-H
eo
«4-< S
o <
a
o
% fe§
m
Q ^ •«
o o< o
o Q< X
o to "9
cS
Q SO a >.
no
a
W '>
.S CO
^ ^0.2®g cS c8 <S3 «J
it u
g t—{
rH ^ > G.1-H >a ^ >a >>
0)
'^ o X CO
3 >* o cx
1— 6 ^ ^ «5 eo -^
90 l>
«>»«»<
O
Tji .-I
»oi-HCMeooseo
t» t-
9<ot»oooeoO'-i 2
5 O a .2
«&
c
o
>^ w P5 O 08
cri
o "O
a-B-
to » •
-
»
-
»
-
^'?,
pa V >>
r-i l^; 3 o »o >o »c »o »o O w 00
eg r!5 o to -^a eo
P
C/J
:z; tf hJ a
0)
H
H
C/5
>1
a
V
O CC
U o
^ ^
I-H
QO a
O
'«*<
g'a
•^
1 a C^
a £
o I a u M > a
;z; V V
o a,
v .2 x
W) CU
fl
bO bO be &C 00 be be
a a a a a a a 60
a >^c^QQQ^^ncg
^
ILLUSTRATIVE PROBLEMS 351
Problem 95*
'There will be found slight discrepancies in the figures given in this problem, doe to the rounding
decimab.
off of
352 ILLUSTRATIVE PROBLEMS
Schedule 1
13,231 "
Schedule 2
OPERATING EXPENSE FOR PERIOD
Wages
Superintendent $ 325.00
Assistant Superintendent 200.62
Foremen 564.12
General Help 1,865.44
Depreciation on Buildings 405.67
Depreciation on Machinery and Equipment 2,502.75
Depreciation on Auto 38.84
Repairs 912.29
Material and Supplies 300.32 $7,115.05
Total $8,697.81
Solution to Problem 95
SILK-THROWING MILL
OPERATING EXPENSE PER POUND OF SILK SHIPPED
Operating
Multi- Operating
Ship- Weights Product Expense
plier Expense
ments per Pound
(1) (2) (1X2) (3) (4) (4-^1)
Tram, 28 /30 Japan Single 546 lbs. 1 546 lbs. $.04064967 $ 22.19 $.040650
Tram, Canton on Cops. . . 1,114 " 6 6,684 •'
271.70 .243896
1,472 " 8 11,776 " 478.69 .325197
Crepe on Bobbins 3,508 " 18 63,144 " 2,566.78 .731693
Crepe on Cops 6,591 " 20 131,820 " 5,358.45 .812994
" "
Total 13,231 213,970 $8,697.81
Problem 96*
From
the data contained in the following schedules,
prepare a statement showing the cost per pound for
winding tram, organzine, and crepe in the winding
department of a silk-throwing mill.
•There will be found slight discrepancies in the figures given in this problem, due to the rounding off
of dedmals.
354 ILLUSTRATIVE PROBLEMS
Schedule 1
Schedule 2
DETAILED ANALYSIS OF LABOR AND EXPENSE
FOR PERIOD
Labor
Tram:
Japan $ 118.89
Canton 182.88
28/30 Japan Single 15.74
Organzine 41.07
Crepe •.
. 712.06
General
Rubbing 11.97
Handing Out Silk 47.61
Soft Silk 48.75
Expense:
Repairs 184.29
Total $1,363.26
Schedule 3
PRODUCTION FOR PERIOD
Tram 1,660 lbs.
Organzine 1,272 "
Crepe 10,099 "
Solution to Problem 96
SILK-THROWING MILL
UNIT COSTS— WINDING DEPARTMENT
Problem 97*
From
the data contained in the following schedules,
prepare a statement showing the cost per pound for
•There will be found slight discrepancies in the figures given in this problem, due to the rounding
off of decimals.
356 ILLUSTRATIVE PROBLEMS
Schedule 1
Schedule 2
Expense:
Repairs 1,818.28
Total $5,398.40
Schedule 3
PRODUCTION FOR PERIOD
Tram 1,660 lbs.
Organzine 1,272 "
Crepe 10,099 "
Solution to Problem 97
SILK-THROWING MILL
UNIT COSTS— SPINNING DEPARTMENT
Organ- Sub- Grand
Tram Crepe General
zine Total Total
Problem 98*
From the data contained in the following schedule
and the cost accounts taken from the books in a silk-
throwing mill, prepare a statement showing the cost of
each product for the period and also an analysis of sales.
Schedule 1
•There will be found slight discrepancies in the figures given in this problem, due to the rounding
off of decimals.
'358 ILLUSTRATIVE PROBLEMS
H
« - -
;?: a «
n H
H as
I? O
04
X
O *
zfi 00 l-H
w « 50 *>
00
<; o u ^5 II
1
(x O h5
«o
o I
00
l>
fO
ti
1-1 t- o>
O •!•
^1
I % to l-H Eh
H eo 1-1 t-
00 Z
»
1> S
Oi Oi
w H
«
O 1-1
as^
<
PM
o H
;?;
Q
o
g
S
O ;?
u
a
s
CO
CS
U
(3 V V 'S
> aj OS
S*
ILLUSTRATIVE PROBLEMS 359
C5 <»4 00 00
O 9« O 00
C5 CO
o o (X 00
9)
«& 0&
a, OQ
00 '^ en ^ 00
W o 05 « OS
to
-N
*
c eo
< Q ^ o '2
o
e •;; O ;; a ._2 u -^
o
.
^ O jj
Cl ^
< 0) o « o c « o « o o
O
PQ
'O ^ i < u u ^
<
Q
H
o
Q
o P
z
)-4 O
Q
IS
H
-a
a
to ;
^^tf
360 ILLUSTRATIVE PROBLEMS
f 00 «o ,^ o
I- 00 O *
O) o (X to
eo * ao
eo
»o :^ t* «o cc
«&
9)
m
* '
a *
CM
t^
00 I-I
a> «o O •* t' o OS
o 3 eo ;o
•^ 3 '^^ 3
1=1o S o O
g. « H t> >>
n
O 03 3 3 is o
n n
o 2 ° 2 o <
^,(Li eg8 Oh > 4, 4,
00 * a •
PL, p^l
I
I
O A
eo
l-H 00
O 00 1^ (V
EH t- «0
00 ii 00 «o »-i
;?
«
»
Q
o c
iz;
g
M
CM
c/2
E>3
E>3
h 3
a 5
II 3 T
ILLUSTRATIVE PROBLEMS S61
s
d o ©
*> -
s
to o
EC ft —
CO
o
"A
s S o =
Q g .2
.U< u .2
.4J ^^ *J O §.2
w a ^ S- « a "
< 2
9 "« §^
^ w V O P5 o
en
o g o
M Co t4
o o
pq n 00 Jt
O
< (M
<
Q
o
^
362 ILLUSTRATIVE PROBLEMS
t~ <3^
05 i~i
05 ,
00
o o
«» ^
I» CO 03
-« o — a< 00
«o o * a» 00 ^ ^ »> o
•«ifi
^^ i-(
-^
t» 00 O 05 ^:
«5 a a rt"
.S <w
•-T *^ w 73 to
9 S a o ° o ° o a o *
a ^ fl
.2 * .2 •^ ca U -^ .2 .2
'*» («W.2
-js « CU (J
^J3
g § .a 3 3 3 .23 n a 3
o^oaoDowo
o o o
CO
o
so 2
J-i
a o V O V o
Ph hol, 6oCli «pLi y eu Oh
~\ Oh V Ph V
Co t-i fcn Ui
00
O O U CJ U
so
m
ILLUSTRATIVE PROBLEMS 363
^30
22 o t-
1-H 00 ^ n?
2 '-'
"-•
"*
^
5 in w
H CO o -* <M 00
V •^ l> O t-
* o
to •«
^" 05"
a .9
15
a a c a -g a
.2 -2 .2 rj .2
c o S g
.2 «3 .2
^ *^
0-0
•
e3 •43
o H t;
^ 3 .5 3 5 3
a °
Oono4>04JO CO O p o
-3 'n -a
o a O 4>
-v
O
V 0^
o ;j o o
a <
o
O O
GO
364 ILLUSTRATIVE PROBLEMS
Solution to Problem 98
SILK-THROWING MILL
DETAILED ANALYSIS OF COSTS FOR PERIOD
28/30 JAPAN SINGLE TRAM
CREPE ON BOBBINS
i
366 ILLUSTRATIVE PROBLEMS
c •*
a
s
^ o»
sb
t»
>o
o
00
a
"3
«
00
ff)
00
«
(M «j>
1
-
1 3 §1
OJ «>
T3
2 2
a
h4 3 5 S
§
5
§ §
o» S 5
«o o o* 00
CO
*
o m OD
S § "3 t^ o
« 00
«s of l>
<N O
lO
i
00 «5 S
00
1 ^X
o !X!
I-H
46-
l>
"
tf m
w >< B
CO
O 00
•*
CO ^CO
•* CO
H hJ 3 I-
•*
o
a
>0 CO
00
00
»»
>o
p
S
a
w
00
'-
i
1
<«1 QB (M O >0 00
.2
1— < 1 -^^H a
cc g g s s s O
M 00 a 00 00
1
g5S5§ »J p-T lo" a»
o
SeS^
i
en
2 2 2 § S
a S ;: ^ S S
s «- «- «- co- 00
(2
a r
2
C
c
C
cc ,
c 1
g
f 1 1
5
C eg e B
c = C
c
P c .
i . (
s c
ILLUSTRATIVE PROBLEMS 367
CREPE ON COPS
Problem 99
The Department of Street-Cleaning operates a
stable where the horses are cared for, a mechanical
department where the mechanics are located who make
the necessary repairs to the buildings and equipment,
and a storehouse from which articles used in the va-
rious divisions of the department are issued. The city
is divided into street-cleaning districts in charge of
superintendents, and the districts are subdivided into
sections in charge of foremen. After the streets have
S68 ILLUSTRATIVE PROBLEMS
Schedule 1
OPENING INVENTORY
Initial Cost
Land $500,000.00
Buildings 750,000.00
Equipment 600,000.00
Stores 100,000.00
Repair Jobs in Process 25,000.00
Schedule 2
FIXED CHARGES
It is estimated that the depreciation
on buildings amounts to
3% and on equipment to 10% a year. The loss to the city by
reason of exemption of land and buildings from taxation amounts
to $1.84 per $100. Interest amounts to 4% on the initial cost.
Schedule 3
PURCHASES
The purchases for the year were as follows:
General Administration $ 25,000.00
Repair Division 10,000.00
Sweeping and Cleaning 50,000.00
Carting and Stable 40,000.00
Final Disposition 200,000.00
Snow Removal 10,000.00
Schedule 4
STORES ISSUED
The stores issued for the year were as follows:
Schedule 5
PAY-ROLL DISTRIBUTION
The pay-roll for the year was as follows:
Schedule 6
GENERAL ADMINISTRATION
The general administration expense of the Street-Cleaning
Department for the year is to be distributed on the following
bases
100%
Schedule 7
REPAIR DIVISION
The repair division reports work done as follows:
100%
Schedule 8
WORK UNITS
The number of yards estimated to be covered by each unit
is as follows:
coc o
ooc oo o o
o oc c c Ovjaxs xjbffeo XO«5
ooox
"3
ooc
o_
d
qq
•od
q
d d
999 =
dd c
lo
OS^IV
xj'Sd
<v&^—
t-H •'^ t-T
o o
«* »o o t-^-* IV ""iv sS-s
9, " 00, S ^'o»
11 «st~"c
o* m 0- —'ot .->"
X*d«5''
-I
x"
» iv"
a»
—
05
oT
— —
« «rs-
c;
» «& » » »
rl
OB-
co"
c ooc
ooc oq oo
qo
o
q o OOqqc
q
OC §ss§ o^s>m <v
coo«o
on
.2 q MJ ddc
^•^ o o'c
ooc d «o d d d "C! X So'
X o oc X 2t-" IV «5
O "OC o o '"IV
«o o_ 9. ". 9 9 =« — X_ o o
e| OOiC
o — cc j>
o •* o" o'
a< oo« 1- «5 p-i
•*
t~
oT
l-H
5t*
#
OS
a S'"
o<
9& ae- «&
« IV
a&
O*
«»
IV
«»
ooc
ooc oo o
oo o o
q o o o oc
qqoc O vCfl
Otf-lV Ss§S •O O 0-
ooc q -2d "igs
^ ^ ooc ^ d d d *o d d c «>2«>
— gd§
•o
« '^ *
•=-o3
r5 « o_>o c >o «o
t-
o l>
o 3* o o c
o
<V M «5 "^ 00
lO
9.SaJ
o" ofc t,"
t-
o)*-*
— IN F-T
— •o
« no'c'ic
X * io<* x"
— oT
o*
O
i^
00*
"o
00 ^-
cs •*
o* -^
«&
w 00
96- 9^ aa
IV
<&
IV «»
«&
fl»
ooc
ooc O o
ooo o
o Q
o o ooc
qoqc §£Sg 0f,0>0 •o o oo
t-oSj
SP t« q «5U^<V
£
oras
e o'o'c
ooc d
o ^^ d ^ dd d c t~ s •* — «=t-
«5 o'"
o_o_c
lO "5"
o
C o"
0*0
o» x"
t-__
at
o*
«
oJ
a; c"
q o_ c
d cC « *
IV
* 9.
d^
" '^.29&
oT o9.
en U l> t- oc «
« 00 «5 "O
(X «*
oo
X l~ —
OS
00 «5 a*
OS
9& " 9& «» «& e& fl&^_ »
r^ H
Mh r/5 oo oo
oo o oo o o oooc
qoqc S6§g o
< O .t
. c
o do'o
ooc
q
d
99
dd
9
d
q
^ dddc doi^
•o
0^ U «'S
o-- o o_o o
o
ic o
a»o
o
« ^ •oooc
C5 oo_o
lo — as
OS X
t^
^
X
M o > o >oo «r
n
e^'d" of i> t-'o"dc
— — -fU! t~ •* CM
Q ptH •O t- CO
» » »
OT
o ooo
ooc oq oo
99
o
®
ooo
qqo §ggi Oo
Pi ooc ^ od o d ddc ddc *o •o
t"
I— O c
<v
".9
>o
1 SSc oSoc
;?^
W e.S «
Wt3
<ot-*o
<»»
*Q
95 «
»o
o*"
OS
r-"(r> -T <»*— 9.
oo'Ms'dc
--IV — X
•*
< ^ <.i.
a;
«e- a& «& » " e&
w w
^ H ooo
ooc o
o oo
99
o
9 qqo oooc
ooo qqqc oq ^
o o
q
u <5
"a^
U
O o' d c
ooo d od d ode dddc d
o o d
Lh H
CT!
OS
o_o_c
o'do
o •« o
o
d
lo o
a*
a«
d
o
o
«
9<
ooc
fl-"-*
t-
r-.
»o o o c
•**
X ui id «
t- «
>o
» «i
OS
»o
oT
r-,-1
"o W5 (C
t-.
•o
x_ — so
00 00 l> 3:
9.
-f
CO s
H «& S- «& » a& «» «-
G
K
M
1
3 V
.£ >
'3
a 1 2
o
OS
-0
J '5 .2
IS u a
-o
1e a
<
e "o "o
c—
is
> tn
^ c , c a in B 1
s .2 ScS c Ca»
1 .2 .2
^E •s c a o ,o ?!
s
c i-
2 2:
^c i s .
c = C
J cE 0. c i
1- e
£ 1 i
372 ILLUSTRATIVE PROBLEMS
Problem 100
Sales 755,849.70
Finished Goods 755,849.70
To charge Sales account with the cost of fin-
Sales 151,169.94
and Loss
Profit 151,169.94
Gross profit on sales.
Raw Material
1914 1914
Jan. 1 Balance . . $ 46,864.26 Dec. 31 Requisitioned $239,461.02
Dec. 31 Purchases 241,249.35 Balance .... 48,652.59
$288,113.61 $288,113.61
1915
Jan. 1 Balance $ 48.652.59
ILLUSTRATIVE PROBLEMS 375
Factory Pay-Roll
1914 1914
Dec. 31 Year's Pay- Jan. 1 Accrued .... I 2,495.34
Roll $377,381.70 Dec. 31 Mfg. Cost . . 360,751.20
Balance .... 3,743.01 Factory Wages 17,878.17
,124.71 ,124.71
1915
Jan. 1 Balance J,743.01
$791,877.54 $791,877.54
1915
Jan. 1 Balance $201,248.55
Finished Goods
1914 1914
Jan. 1 Balance . . . $ 64,968.03 Dec. 31 Cost of Sales $755,849.70
Dec. 31 Prime Cost . 590,628.99 Balance .... 85,568.55
Mfg. Exp. . . 185,821.23
$841,418.25 $841,418.25
1915
Jan. 1 Balance . . . . $ 85,568.55
Sales
1914 1914
Dec. 31 Cost of Sales $755,849.70 Dec. 31 $907,019.64
Gross Profit 151,169.94
$907,019.64 $907,019.64
376 ILLUSTRATIVE PROBLEMS
Superintendence
1914 1914
Jan. 1 Undistributed $ 36,224.76 Dec. 31 Mfg. Exp. . . $112,488.78
Dec. 31 114,300.00 Balance . . . 38.035.98
$150,524.76 $150,524.76
1915
Jan. 1 Balance .... $ 38,035.98
1915
Jan. 1 Balance $ 25,317.06
Manufacturing Expenses
1914 1914
Dec. 31 Overhead ... $ 73,332.45 Dec. 31 Finished
Superintendence 112,488.78 Goods .... $185.821.23
$185,821.23 $185,821.23
Problem 101
Income Average
Sales Cost
100,000 ft. Rough Lumber $ 750.00 $ 7.50
35,000 ft. Dressed Lumber 350.00 10.00
$1,100.00 $ 8.15
Expenses
$1,850.00 $ 6.50
S78 ILLUSTRATIVE PROBLEMS
Less — Inventory (estimated value)
100.000 ft. Logs in Woods at $2.50 . . $250.00
100,000 " Logs at Mill " 4.00 . . . 400.00
50,000 " Rough Lumber " 7.00 . . . 350.00
15,000 " Dressed Lumber " 8.00 . . . 120.00 1,120.00
Cost $ 730.00
Profit 370.00 $1,100.00
{Virginia C. P. A. Examination.)
Hauling to Mill:
" "
Cost of Logs . . . 300,000 3.00 $ 900.00
Hauling " 1.00 300.00
Total $1,200.00
Less —Inventory 100.000 " " 4.00 400.00
Sawing:
" "
Cost of Logs . . . 200,000 4.00 $ 800.00
Sawing " 1.50 300.00
Total $1,100.00
Less — Inventory 50,000 " " 5.50 275.00
Planing:
Cost Rough
Lumber 50,000 " " 5.50 $ 275.00
ILLUSTRATIVE PROBLEMS 379
Total $ 325.00
Less— Inventory 15,000 " " 6.50 97.50
(b) Profit and Loss Account. See Volume III, Chapter IX,
for a description of a Profit and Loss account.
Problem 102
(a)
(b)
Deduct —
Cost of Goods Sold: —
Inventory of Finished Goods on
hand January 1, 1907 (1,600
wheels at $30) $ 48,000.00
Cost of Wheels Manufactured . . 182,000.00 $230,000.00
382 ILLUSTRATIVE PROBLEMS
Less—Inventory at December 31, 1907 (800
wheels at $35) 28,000.00 202,000.00
—
Less Expenses
Agents' Commissions '.
$ 90,000.00
Expense
Selling 30,000.00
Branch Expense 40,000.00 160,000.00
Deduct —
Other Charges:
Loss on Bad Debts 8,000.00
Problem 103
1000 Reserves:
1001 Reserve for Depreciation — Buildings
1002 Reserve for Depreciation —Machinery and Equip-
ment
1003 Reserve for Depreciation —Office Furniture and
Fixtures
1004 Reserve for Bad Debts
1005 Reserve for Freight and Discounts
2000 Sales:
2001-2011 (Subsidiary accounts to be kept for sales of
various styles or types of bicycles and finished parts.)
known.
In this case the method of arriving at an approxi-
mate inventory each month would be as follows:
Total charges contained in the controlling accounts:
Account No. 200 $
Account No. 1500
Account No. 1600
Account No. 1700
Total $
Problem 104*
From the data contained in the following schedules
"
make a calculation of the cost per chargeable or "sold
*There will be found slight discrepancies in the figures given in this problem, doe to the rounding
off of decimals.
ILLUSTRATIVE PROBLEMS 387
Schedule 1
DISTRIBUTION OF RENT
The annual rent paid is $36,060, and is to be prorated on the
basis of productive floor area which isfound to be as follows by
measurement
Proof-Reading 740
Hand Composition 6,817
Keyboard Monotype 288
Casters 634
Plate Finishing and Repairing 346
Job Press Department:
Kelly Presses 834
Victoria and Caxton Presses 417
Universal Presses 1,424
Gordon Press 208
Cylinder Press Department:
Pony Press 1,005
00, 1-Color Presses 3,519
000, 2 " " 4,019
0000, 1 " " 7,168
00000, 1 " " 2,010
00000, 2 " " 2,010
Cutting 1,000
Total 44,150
888 ILLUSTRATIVE PROBLEMS
Schedule 2
DISTRIBUTION OF POWER
The power bill for the month amounted to $1,940.58, and is
Total 81,351
Schedule 3
DISTRIBUTION OF DEPRECIATION
Depreciation is taken at 15% per year on the plant invest-
ment which is as follows:
Plant
Departments and Factory Divisions Investment
Total $565,432.93
Schedule 4
PAY-ROLL ANALYSIS
The analysis of the factory pay-roll for the month appears
as follows:
Total $46,909.45
Schedule 5
ANALYSIS OF DEPARTMENTAL PURCHASES
The analysis of purchases chargeable to departmental ac-
counts is as follows for the month
Departments and Factory Divisions Purchases
Total $6,943.35
Schedule 6
CHARGEABLE HOURS
The chargeable hours for the period are reported to be as
follows
Chargeable
Departments and Factory Divisions Hours
DETAILED ANALYSIS OF
STATIsf
Hand KevWtl
Plate JOB PRESS DEPT. J
Proof Finish-
General Shipping Compo- Casters
ing and Victoria Univer-
Reading Kelly Gordon
sition type Rtpainnj Caxton sal
i. <)4 Vo % %
Floor Space in Sq.ft. 740 l.?7 ^15.43 288 .6634 144346 .78 834 1.89 4n .M I4Z4 Vh 208 .ft
Horse-power Hours 4B .51 623 .76 456 .56 646 .79306 .38 375 .46 48 .05
Plant Investment 79146752 4{i0610i 5548.00 2134181 1,836.94 10,125.94 2,541.89 5748.07 462^6
DEPARTMENTAL COSTSJ
1 Pay-roll - Factory 1,408.02 3U 36.06 172306 737a29 411.42 427.79 385i02 79859 286.67 88QS0 81.91
4 Purchases 173581 237.29 332.43 4765 7098 853 eo.o3 I6i36 59.96 &19
5 Rent 15.64 5020 46384 19.54 39.38 22.47 56.81 2876 96l79 14.12
7 Depreciation e 15% v. 99335 57578 44.35 291.78 2296 I26i57 31.77 71.85 5.79
8 Total 11.672.23 2i373.35 1,27^26 ai75a34 532.81 844.78 449.86 1,05739 37035 1,118.04 108.15
11 Total U672.23 a3B35 6184723 606.91 94705 44986 1.05739 370.35 1.1 18.04 108.15
CDistrof GenlandShip'g 16.88% -104% 1.62% .77'/. 1.81 •/. j64% 1.92 % .19 %
13 on a Rercentage Basis 2,53969 156l47 243.74 115.85 27^32 96.29 288.87 2859
Lt Total 11 and 13 1^^386.92 76338 U90.79 56571 1.329.71 466i64 1,406.91 136.74
Problem 104
TICS
CYLINDER PRESS DEPT. WEBB PRESS DEPT. SLIP SHEETING
Wrapping
Pony
oo oooo ooooo ooooo 64P.P 32 PR 32PP 64PP Cutting
Job Total
l-Col. 2 -Col. 1-Col. l-Col 2-Col. 2-CoL 2 -Col. l-Col. l-Col.
Job Cylinder
V. V. V.
ID05?.27 ^19 797 4019 aio 7168 16.3 3)10 455 g)l0 455 5213 11.80 I?99 2.94 3896 &82 ^3 2X IPOO 226 44,150 100
81 .10 1806 122 ^0 757 61)96 7.49 I{t63 180 i8M4.77 33,995*8Z 4.973 6.11 1L0ZO-Q55 5035 9.87 974 1.19 81,351 l2o
1286.92 17338.60 44,770.30 43,178.96 IL703.37 24.890.25 I72.94I.58 ZHQSa 6Q75762 20534.41 292342 565432.93
82 552 977 U572 376 519 2P34 605 1289 454 354 104 124 354 18,624
2,752.75
4.766i6
2198 16a82 295.99 49359 112.64 151.19 1.219.03 28727 1,299.16 304.00 2145 6.94335
68.23 239.58 273.55 489.98 136.77 l36iTr 354.81 88.28 265.13 88.65 5620 3OOS.OO
1.93 4108 14^94 I4&4I 34.89 92.55 810.79 118.63 262.87 191.69 23.46 I.94Q58
16.09 219.23 55a63 539.11 146.29 311.13 1.536.77 410.24 759.47 369.18 36.54 7.06788
297.57 1,880.40 3524.16 5i4l4.85 1,179.94 1.909.34 13,465.3 3,736.59 8.62068 3,06749 509.13 34.15 7&96 116.72 7^38567
297.57 l,88a40 i524.16 ^414.85 1,179.94 1,90934 13465.13 3173659 &62068 3067.49 509.13 34.15 78.96 116.72 73.385.67
31% 3.22% 6.04% a28% Z02% 3.27 V. 3.08% 6.40% 1477% &2S% .87% .06% .14% .22% 100%
76.73 484.47 908.75 1,39&?3 30392 491.99 3.472.52 96291 ^22223 789.89 130.90 9.03 21.06 3313
374.30 Z364.87 4,43Z9I 6^81108 I.483l86 Z40U3 I6i93765 4,69150 K1842.9I 3857.38 640.03 43.18 100.02 14985 73,38567
»4.56 *4.28 *4.54 *4.07 *a95 *4.62 *8.33 7.77 *3.4! *8.50 l.6f *.42 *.8I *.42
394 ILLUSTRATIVE PROBLEMS
Cutting 354
Slip Sheeting —Job Department 104
Slip Sheeting — Cylinder Department 124
Wrapping —Job Department 354
Total 18,624
Schedule 7
DISTRIBUTION OF PROOF-READING DEPART-
MENT LABOR AND EXPENSE
The charges to the proof-reading department will be found
to amount to $1,273.26 (pay-roll $1,223.06, and rent $50.20),
which amount is to be distributed to the hand composition,
keyboard monotype, and casters departments on the basis of
the chargeable hours in each.
Schedule 8
Schedule 9
Problem 105
THE AMERICAN REFINING COMPANY AND
ITS CONSTITUENT COMPANIES
Assets
depreciation $ 45,931,123.93
Investments— General 24,782,540.68
396 ILLUSTRATIVE PROBLEMS
Investments — Insurance Fund 9,500,000.00
Investments— Pension Fund 1.750,000.00
Merchandise and Supplies, including raw and refined sugar,
syrup, material in process of manufacturing boneblack,
cooperage, and other stock and supplies on hand 9,142,074.71
Prepaid Accounts, Insurance, Taxes, etc 309,051.18
Loans 1,121,266.10
Accounts Receivable 3,322,489.23
Accrued Income, Interest Earned, and Dividends Declared
but Not Yet Collected 1,047,043.91
Cash on Hand, with Trust Companies, Banks, and Short-
Term Loans 40,493,252.19
$137,398,841.93
Liabilities
Capital Stock:
Preferred $45,000,000.00
Common 45,000,000.00 $ 90,000,000.00
Sundry Reserves:
For Insurance $ 9,500,000.00
For Pension Fund 1,750,000.00
For Improvement of Plants 3,367,514.84
For Trade- Mark Advertising 2,000,000.00
For Contingencies 823,647.99 17,441,162.83
$137,398,841.93
ILLUSTRATIVE PROBLEMS 397
Credits
$2,991,465.39 $ 9,756,379.42 $10,055,291 41
880,609.09 792,990.70 1,006,002.25
2.312,646.21 2,905,737.10 3,129,948.70
Net Profit from Investments 248,336.34 21,544.85
Debits
Depreciation, Renewal, and Replacement $ 790,304.71 $ 2,000,000.00 $ 2,000,000.00
481,906.98 3,383,562.09 4,000,000.00
6,299,972.00 6,299,972.00 6,299,972.00
BALANCE SHEET
For Years 1918, 1919 and 1920
Assets
$ 48,763,560.47 $ 47,246,442.89 $ 45,931,123.93
Investments — General 22,577,772.00 23,972,036.34 24,782,540.68
8,000,000.00 9,000.000.00 9,500,000.00
1,000,000.00 1,250,000.00 1,750,000.00
16,963,384.52 18,654,839.97 9,142,074.71
252,834.04 1,527,643.32 309,051.18
3,803,274 90 1,222,193.00 1,121,266.10
Accounts Receivable 4,607,398.09 3,833,259.72 3,322,489.23
Accrued Income 468,844.67 555,907.03 1,047,043.91
Cash 15,624,806.32 22,717,453.53 40,493,252.19
Liabilities
Capital Stock $ 90.000,000.00 $ 90.000,000.00 $ 90,000,000.00
10,137,705.62 13,475,267.87 17,441,162.83
Acfounts and Lo.ins Payable 3,999,462.92 6,555,963.24 8,097,115.45
Dividends Declared and Outstanding 1,595,904.25 1,599,833.00 1,599,036.75
16,328,802.22 18,348,711.69 20,261,526.90
Questions
Problem 106
THE B F COMPANY
CONDENSED BALANCE SHEET
December 31, 1919
Assets
Capital Assets:
Real Estate, Buildings, Plant, Machinery, and Sundry
Equipment less Reserve for Depreciation $12,679,151.72
Patents 583,650.00
Good- Will 57,798,000.00
$71,060,801.72
Investments in other Companies, etc 1,197,058.00
Soci6t6 Fran^aise B F —representing the net investment at
December 31, 1919 570,987.32
20,587 shares of 7% Cumulative Preferred Stock in Treasury
at par 2,058,700.00
Current Assets:
Inventory of Raw Materials, Partly Manu-
factured and Finished Stock $12,614,926.67
Trade Accounts Receivable after deduct-
ing- Reserve to cover Doubtful Ac-
counts, Discounts, and Allowances .... 4,699,938.10
Other Accounts Receivable 777,266.85
Bills Receivable 586,274.70
Cash in Banks and on Hand 723,053.50 19,401,459.82
$94,511,956.87
Liabilities
Capital Stock:
600,000 shares Common Stock, $100 each $60,000,000.00
300,000 shares of 7% Cumulative Pre-
ferred Stock of $100 each 30,000,000.00 $90,000,000.00
300,000.00
Surplus:
Balance at December 31, 1918 $ 806,235.24
—
Add Net Profit for the year ending
December 31, 1919, as per annexed
statement 2,599,747.39
$ 3,405,982.63
$94,511,956.87
THE B F COMPANY
PROFIT AND LOSS ACCOUNT
For the Year Ending December 31, 1919
$ 3,549,429.26
Deduct:
Reduction of Treasury Preferred Stock from
Cost to Par Value $168,417.03
Provision for Depreciation 541,358.09
Interest on Bills Payable 239,906.75 949,681.87
REPORT OF ACCOUNTANTS
February 17, 1920.
Questions
proprietor's salary, 97 61
showing net profit, 88, 97 Classification of accounts, 100
Branch stores, Interstate Commerce Commission,
accounts kept at head office, 119 railroad accounts, 215
incorporation of proprietorship manufacturing concern, 382
business, 253 Closing entries (See "Entries,
interbranch profits, 108 closing")
inventory, valuation of, 108 Combinations and consolidations,
opening entries on incorporation, 277-328
140, 253 acquirement by stock ownership,
profit and loss analyses, 119, 253 328
treatment of contingent profits as by purchase, 202, 277, 281, 288
deferred credit, 106, 108, 119 cash entries for, 277
trial balance, general, 119 opening entries for, 202, 277
plant and sundry assets account,
use of, 277, 281
vendee company, opening en-
Cancellation of bonds, 152, 166 tries, 288
Inventory Continued M
analysis of, 23
estimating, after loss, 127, 130, Manufacturing concern,
331 classification of accounts, 382
gross profit on, 127 statements, 221, 228, 231, 233
invoices not received, how handled, Mercantile business, statements of,
90 242, 249, 253
proof of loss, after burglary, 331 Merchandise,
statement showing differences, 336 account,
testing, method of, 336 adjustment after fire loss, 196
valuation, branch stores, 108 analysis of, 23
closing, 23
inadequacy of, 23
gross profit on, ascertaining, 127
Job order cost system, 386 inventory (See "Inventory")
Job order vs. process costs illustrated, Mortgage, interest on, how secured,
347. 351 statement of affairs, 66, 70
N
Ledger, subsidiary, reconcilement of, Net profit, calculation of,
with controlling account, 11 invested capital, 341
Liabilities, single-entry statements, 88, 97
accrued, Net worth, showing of, on statement
adjusting entries, 180 of affairs, 61
consolidated balance sheet, treat- Notes receivable,
ment on, 328 discounted, 61, 70
effect on surplus, 180 consolidated balance sheet, treat-
necessity for recording, 215 ment on, 319
realization and liquidation state- contingent liability, 345
ment, treatment of, 83 journal entries, 13
salariesdue but not recorded,
132
contingent,
journal entries to record, 178 Opening entries, 10 (See also "Entries,
notes receivable as, 345 opening")
Liquidation and realization (See Organization expense,
" Realization and liquidation capitalization of, 207
statement," "Statement of consolidations, how treated in, 304
affairs")
receivership transactions, entries
covering, 266
surplus adjustments, 266 Partnership,
Loss, distribution of, deficiency ac- accounting, 27, 56, 140, 183, 186,
count, 61 187, 190
410 INDEX
Partnership Continued Preferred claims, statement of affairs,
acco unti ng Co ntinued 61
admission of partner, 183, 186, Premium,
187 bonds in sinking fund, 300
balance sheet adjustments, 45 stock (See "Stock")
capital account, adjustments Process cost,
affecting, 116, 183, 187 accounts showing, 357
closing entries, on incorporation, analysis of costs, 357
56, 140 compared with job order costs,
distribution of profit and loss, 347, 351
27, 31, 183, 187 per unit of product, 353, 357
intereston capital and drawings, Profit and loss account, 399
investments on consolidation, adjusting entries, 132
319 (See also "Consolidated as surplus account, 266
balance sheet") branch stores, 119, 253
post-closing trial balance, 45 railroad, 215
profit and loss statement, 45, showing cost of production, 377
190 Profit and loss statement,
profits, adjustments affecting, comparative, 395
190 partnership accounting, 45, 190
single-entry statements, opening preparation of, from single-entry
entries, 88 records, J12 (See also "Book-
statement of partnership ac- keeping, single-entry")
counts, 140 reconcilement with balance sheet,
dissolution, 34, 35, 37, 39, 42. 190 132, 337
deficit offset with loan, 34 Profit or loss, single-entry bookkeep-
distribution of assets and loss, ing (See "Bookkeeping, single-
34, 35, 42 entry")
good- will, accounting for, 56, 140 Profits,
life insurance policy, accounting accounting for, after fire loss, 199
for, on death of partner, 116 contingent, in branch accounting,
overpayment of partner, pre- treatment of, 106, 108, 119
venting, 35, 37 contracts not realized, treatment of,
payment by instalments, 35, 37 293
salary of partner, how treated, 42 division of partners', 27, 31, 183,
good-will, 187 (See also "Partnership ac-
sale of, 56 counting")
valuation 190
of, 183, 186, 187, effect of depreciation on, 132
incorporation, method 56
of, gross,
Percentage of diminishing value determination of, 23
method of calculating deprecia- merchandise, 127
tions, 168 sales, 341
Petty cash fund, 7 sales, cost of, 341
Plant and sundry assets account, 170, instalment sales, entry recording,
202, 277, 281 109
INDEX 411
—
Surp lus Continued errors,
adjustments, carried in suspense account, 137
accrued liabilities, 180 correction of, 319
applicable to prior periods, 125, location of, 343
132 parent and subsidiaries, 304
combinations and consolidations) post-closing, in partnership ac-
284, 312 counting, 45
depreciation entries, 15 16, 18. Trustee, trading account, 78, 83
21 Trusteeship (See " Realization and
errors affecting, 180 liquidation statement")
fire loss, 194 Turnover, computation of rate of,